Maternal, Fetal, and Placental Physiology and Pathology Flashcards

1
Q
  1. Compared to the nongravid state, which compound is LEAST likely to change during pregnancy?
    A. Pre-Beta-lipoprotein
    B. Cholesterol
    C. Free fatty acid
    D. Insulin
    E. Glucagon
A

E. To ensure a persistent source of metabolic fuel to the fetus, there are several vital changes in maternal hormones and feto-placental unit. Pre-β-lipoprotein, a very-low-density lipoprotein that normally represents a very small percentage of total lipoprotein, is increased in pregnancy. High-density lipoprotein (HDL) cholesterol levels increase in early pregnancy, whereas low-density lipoprotein (LDL) cholesterol levels increase later in pregnancy. The metabolic changes of pregnancy include hyperinsulinemia, insulin resistance, relative fasting hypoglycemia, increased circulating plasma lipids, and hypoaminoacidemia. During pregnancy, glucagon does not play a significant role as a diabetogenic factor and does not change appreciably.

How well did you know this?
1
Not at all
2
3
4
5
Perfectly
2
Q
  1. Human placenta is an incomplete steroidogenic organ, and estrogen synteshis by the placenta requires which of the following?

A. Dehydroepiandrosterone sulfate (DHEAS)
B. Cholesterol
C. 17-alpha-hydroxypregnenolone
D. Trophoblast cytochrome P450 (CYP) enzyme
E. Tumor necrosis factor-alpha (TNF-alpha)

A

A. In human parturition, maturation and activation of the fetal hypothalamic-pituitary-adrenal (HPA) axis is an important component. The activation of the axis is linked with increased production of dehydroepiandrosterone sulfate (DHEAS) from the intermediate zone of the fetal adrenal. DHEAS is directly aromatized in the placenta to estrone, and it can also be 16-hydroxylated in the fetal liver and converted in the placenta to estriol (16-hydroxy-17β-estradiol). This occurs because the human placenta is an incomplete steroidogenic organ, and estrogen synthesis by the placenta requires DHEAS as a steroid precursor. Cholesterol is the precursor to pregnenolone which is converted to progesterone in the placenta by 3-β hydroxysteroid dehydrogenase. Pregnenolone cannot be converted to 17 α-hydroxypregnenolone because the placenta lacks 17 α-hydroxylase. Among sheep and cows, parturition involves activation of the fetal HPA axis at term. In such animals, a sharp rise in the concentration of adrenocorticotropic hormone (ACTH) and cortisol in the fetal circulation occurs 15 to 20 days before delivery. This results in increased expression in the placenta of the Trophoblast cytochrome P450 (CYP) enzyme 17α-hydroxylase/C17,20-lyase (CYP17), which catalyzes the conversion of pregnenolone to 17α-hydroxypregnenolone and dehydroepiandrostenedione. The resultant fall in progesterone and rise in estrone and 17β-estradiol levels in the maternal circulation stimulate the uterus to produce PGF2α, which provides the impetus for labor in such animals. Creasy and Resnik’s Maternal-Fetal Medicine: Principles and Practice, 7th Edition: 68-9.

How well did you know this?
1
Not at all
2
3
4
5
Perfectly
3
Q

A 34 year old G1P0 female at 37 weeks gestation with decreased fetal movement andis diagnosed with fetal death. Which of the following is the most appropriate test in the evaluation?

A. Antinuclear antibody testing
B. Syphilis serology
C. Factor V Leiden mutation testing
D. Bile acids
E. Placental pathology

A

E. Stillbirth is defined as pregnancy loss that occurs after 20 weeks of gestation. In some cases, a cause is never identified, but using a comprehensive and thoughtful evaluation system proposed by the Stillbirth Collaborative Research Network yields a “possible or probable” cause of death in 76.2% of cases. The first part of the evaluation is a clear, thorough medical and obstetric history. This will guide the remainder of the evaluation; some testing is not routinely recommended, but a clue in the history may point a provider toward that test in a given patient. In all cases of stillbirth, the recommended tests are:
● -Fetal postmortem evaluation
● -Placental pathology
● -Karyotype or microarray if congenital anomaly/karyotype failure
● -Fetal-maternal hemorrhage testing (before induction of labor)
● -Antiphospholipid antibody testing
Any additional tests should be done only if there are indications from the history. This is true in particular of infectious workup, which should not otherwise be performed routinely (TORCH titer testing). Inherited thrombophilia workup is also not routinely indicated in cases of stillbirth.

ACOG Practice Bulletin 102: Management of Stillbirth.

How well did you know this?
1
Not at all
2
3
4
5
Perfectly
4
Q

. Which of the following conditions is most likely to result in a placenta that is small for gestational age?

A. Hydrops fetalis
B. Type II diabetes mellitus without end organ damage
C. Placenta previa
D. Maternal renal artery stenosis
E. Syphilis

A

D. Placental pathologic examination is an important component of the evaluation of stillbirth and many other obstetric and maternal/fetal disorders. In the hands of a trained pathologist, the gross and histologic examination of this unique organ can yield many diagnostic clues. A small for gestational age placenta, defined as <5% of expected weight for gestational age, can be seen as a result of many conditions that have a common origin of reduced uteroplacental blood flow leading to impairment of villous growth and development. Maternal conditions that cause these findings include hypertension and hypertensive disorders of pregnancy (preeclampsia, eclampsia) as well as diabetes mellitus with accompanying renal disease. Other conditions that result in impaired placental growth are aneuploidy and chronic infection. There are some conditions that cause large for gestational age placenta, defined as (>95% of expected weight for gestational age. Maternal diabetes (without renal disease as noted above), hydrops (both immune and nonimmune), and certain infections such as syphilis are known to cause enlargement of the placenta. Although placenta previa is listed as a possible answer, abnormal placentation is not associated with an increase or decrease in expected size.

Creasy & Resnick’s Maternal-Fetal Medicine: Principles and Practice, 7th edition, Chapter 45, p. 726.

How well did you know this?
1
Not at all
2
3
4
5
Perfectly
5
Q
  1. Which of the following is MOST likely to be deficient in a pregnant woman who adheres to a vegan diet?

A. Calcium
B. Vitamin D
C. Riboflavin
D. Vitamin B12

A

D. Precise data about the effect of vegetarian or vegan diets on perinatal outcomes are sparse. In women who are ovo-lacto vegetarians – that is, those who include eggs and/or dairy in their diet – there may be no deficiency. However, vegan women are most likely to have B12 deficiency because the primary source of this nutrient is animal food sources. These women should have their B12 levels assessed, and if low, encouraged to have B12 supplementation. This can be accomplished by incorporating foods such as meat substitutes or soy milk that are fortified with B12 by the manufacturer, or by taking injectable or oral supplementation. The other nutrients listed as answer choices (calcium, vitamin D, and riboflavin) as well as iron are also at risk of being below normal in vegan women, and should be monitored. These nutrients are readily available in the right plant based foods, and these women should be counseled to include those sources and/or other supplementation to ensure adequate levels.

How well did you know this?
1
Not at all
2
3
4
5
Perfectly
6
Q
  1. A 25 year old multigravida woman presents for her obstetrical visit at 27 weeks of gestation. She complains of headache and insomnia for the past 4 months, as well as episodes of diarrhea with up to 4 loose stools per day. On physical examination,you note bilateral proptosis, and her outstretched hands have a fine tremor. During the evaluation of her neck, there are no palpable masses, and the thyroid gland does not appear to be enlarged. Laboratory evaluation demonstrated a serum TSH of .8 mIU/L (elevated) in association with a serum total thyroxine of 15.1 mg/dL (elevated). Which of the following is the MOST likely diagnosis?

A. Graves disease
B. Toxic thyroid adenoma
C. TSH-producing pituitary tumor
D. Toxic multinodular goiter
E. Chronic thyroiditis

A

C. The patient exhibits the clinical signs of hyperthyroidism. The laboratory results suggest a thyroid-stimulating hormone (TSH)-producing pituitary adenoma consistent with excess TSH secreted from the pituitary (normal TSH in the second trimester is 0.2-3.0 mIU/L) and excess thyroxine secreted from the thyroid (normal total thyroxine in the second trimester is 7.5-10.3 mg/dL), with no feedback inhibition. TSH-secreting pituitary adenomas are rare, benign tumors that account for approximately 1% of all pituitary adenomas. These tumors produce excess TSH, which causes the thyroid gland to enlarge and produce thyroid hormone in excess, leading to clinical hyperthyroidism. Patients typically exhibit signs and symptoms of hyperthyroidism but may also present with headaches and visual field loss. Early diagnosis and correct treatment (generally surgical resection) optimize prognosis. In Graves disease, toxic thyroid adenoma, and toxic multinodular goiter, the TSH would be suppressed, and there would be an increased amount of thyroid hormone produced, leading to signs and symptoms of clinical hyperthyroidism. In patients with chronic thyroiditis, there is decreased thyroid hormone production from thyroid gland failure, and TSH is elevated.

How well did you know this?
1
Not at all
2
3
4
5
Perfectly
7
Q
  1. A 25 year old multigravida woman presents for her obstetrical visit at 27 weeks of gestation. She complains of headache and insomnia for the past 4 months, as well as episodes of diarrhea with up to 4 loose stools per day. On physical examination,you note bilateral proptosis, and her outstretched hands have a fine tremor. During the evaluation of her neck, there are no palpable masses, and the thyroid gland does not appear to be enlarged. Laboratory evaluation demonstrated a serum TSH of .8 mIU/L (elevated) in association with a serum total thyroxine of 15.1 mg/dL (elevated). Which of the following is the MOST likely diagnosis?

A. Graves disease
B. Toxic thyroid adenoma
C. TSH-producing pituitary tumor
D. Toxic multinodular goiter
E. Chronic thyroiditis

A

C. The patient exhibits the clinical signs of hyperthyroidism. The laboratory results suggest a thyroid-stimulating hormone (TSH)-producing pituitary adenoma consistent with excess TSH secreted from the pituitary (normal TSH in the second trimester is 0.2-3.0 mIU/L) and excess thyroxine secreted from the thyroid (normal total thyroxine in the second trimester is 7.5-10.3 mg/dL), with no feedback inhibition. TSH-secreting pituitary adenomas are rare, benign tumors that account for approximately 1% of all pituitary adenomas. These tumors produce excess TSH, which causes the thyroid gland to enlarge and produce thyroid hormone in excess, leading to clinical hyperthyroidism. Patients typically exhibit signs and symptoms of hyperthyroidism but may also present with headaches and visual field loss. Early diagnosis and correct treatment (generally surgical resection) optimize prognosis. In Graves disease, toxic thyroid adenoma, and toxic multinodular goiter, the TSH would be suppressed, and there would be an increased amount of thyroid hormone produced, leading to signs and symptoms of clinical hyperthyroidism. In patients with chronic thyroiditis, there is decreased thyroid hormone production from thyroid gland failure, and TSH is elevated.

How well did you know this?
1
Not at all
2
3
4
5
Perfectly
8
Q
  1. Which type of cancer is most likely to metastasize to the placenta?

A. Leukemia
B. Lymphoma
C. Melanoma
D. Breast cancer
E. Lung cancer

A

C. Malignant melanoma accounts for approximately 8% of malignant tumors during pregnancy. It is among the most aggressive forms of cancer and represents 31% of metastases to the placenta. If melanoma metastasizes to the placenta there is a 17% chance of it also metastasizing to the fetus. Hematologic malignancies are the second most common tumor to spread to the placenta and fetal dissemination has also been reported. Lung cancer is the leading cause of cancer death among women and is the second most common type of cancer in adult women behind breast cancer. Gabbe’s Obstetrics, 6th edition; pages 1078, 1090-1091.

How well did you know this?
1
Not at all
2
3
4
5
Perfectly
9
Q
  1. A patient presents for assessment of fetal growth at 32 weeks gestational age, and the fetus is noted to have ventriculomegaly and periventricular calcifications. She cares for 2 outdoor cats. Which of the following is the best test for the diagnosis for fetal infection?

A. Detection of toxoplasma DNA in amniotic fluid using PCR
B. Presence of toxoplasma IgM specific antibody
C. Detection of very high toxoplasma IgG antibody titer
D. Documentation of IgG seroconversion from negative to positive
E. Detection of toxoplasma DNA in maternal serum by PCR

A

A. Toxoplasma gondii is a protozoan that is dependent on wild and domestic cats, which are the only known host for the oocyte. It has three distinct life forms: trophozoite, cyst, and oocyst. The invasive trophozoite is released in the intestines of animals, such as cows, that ingest the oocyst. Human infection occurs when infected meat is ingested or oocysts are ingested via contamination by cat feces. Most infections in immunocompetent humans are asymptomatic, and approximately half of all adults in the US have an antibody to toxoplasma. Immunity is usually long-lasting. The frequency of seroconversion during pregnancy is approximately 5%, and clinically significant infection occurs in just 1 in 8000 pregnancies. About 3 in 1000 infants show evidence of congenital infection, but 40% of neonates born to mothers with acute toxoplasmosis have evidence of infection, particularly when maternal infection develops during the third trimester. Clinical manifestations can include a disseminated purpuric rash, hepatosplenomegaly, ascites, chorioretinitis, uveitis, periventricular calcifications, ventriculomegaly, seizures, and developmental delay. Maternal infection is best confirmed by PCR of the maternal serum. While serologic tests for toxoplasma IgM, high-titer IGG, and documentation of seroconversion are suggestive of an acute Infection, detection of toxoplasma DNA in amniotic fluid using PCR is the best test for confirmation of fetal infection. Creasy and Resnick’s Maternal-Fetal Medicine: Principles and Practice, 7th edition, Chapter 51, page 847.

How well did you know this?
1
Not at all
2
3
4
5
Perfectly
10
Q

Which of the following cardiac lesions has the poorest prognosis in pregnancy?

A. Moderate aortic stenosis
B. Hypertrophic cardiomyopathy
C. Unrepaired ventricular septal defect (VSD) with right to left shunt and pulmonary hypertension
D. Marfan syndrome with aortic root diameter 3 cm
E. Unrepaired aortic septal defect (ASD)

A

C. Unrepaired ventricular septal defect (VSD) with right-to-left shunt and pulmonary hypertension is the correct answer. Eisenmenger syndrome is a a congenital heart defect that is characterized by communication between the pulmonary and systemic circulations. The most common underlying defect in these patients is a large VSD. The communication between the higher pressure system of the left, and right ventricle leads initially to a left to right shunt of blood toward the pulmonary vasculature. The increase in the pulmonary vasculature over time leads to pulmonary hypertension and hypertrophy. Eventually, right-sided pressures increase to maintain perfusion and ultimately lead to a shunt reversal resulting in a right to left shunt. Pregnancy carries a 50% risk of mortality in these patients, though newer data places the risk at closer to 25%. In women who survive there is an increased risk of poor outcomes for the fetus as well. Sudden death can occur at any time; however, labor, delivery, and immediately postpartum carry the highest risk. Management during pregnancy focuses on maintaining pulmonary blood flow and avoiding hypotension. Times of greatest risk during labor include administration of regional anesthesia and postpartum hemorrhage. Women with Eisenmenger syndrome tend to tolerate normal blood loss; however, postpartum hemorrhage and the resulting hypotension are poorly tolerated. These patients need a planned delivery with intensive monitoring and multidisciplinary care. Aortic stenosis in reproductive-aged women is most often due to a bicuspid aortic valve. Stenosis worsens over time due to calcification and restriction in leaflet motion. Women with mild to moderate aortic stenosis tolerate pregnancy well. Severe (or “critical”) aortic stenosis is not well tolerated during pregnancy and can be life-threatening. The underlying mechanism of adverse outcomes is related to a fixed cardiac output. Throughout pregnancy, there are multiple factors that lead to decreased preload therefore further worsening a fixed cardiac output. This includes regional anesthesia and hemorrhage. The maternal mortality rate associated with severe aortic stenosis is 8% with the greatest risk to women whose valve gradient exceeds 100 mm Hg. Hypertrophic cardiomyopathy is an autosomal dominant condition that causes cardiac hypertrophy, myocyte disarray, and interstitial fibrosis. In addition to ventricular hypertrophy, there may or may not be left ventricular outflow tract obstruction. Common symptoms are dyspnea, chest pain, syncope, and arrhythmias. In patients with outflow tract obstruction, there is worsening during pregnancy due to the normal decrease in peripheral vascular resistance. Valsalva during labor can result in an increase in outflow tract obstruction and should be minimized. Sudden death can occur due to complex arrhythmias. Marfan syndrome is an autosomal dominant condition with variable expressivity. The condition is characterized by a defect in connective tissue particularly fibrillin. This weakness can especially be seen in the aorta increasing the risk of complications such as aortic dissection. Women whose aortic root is >40mm are at the greatest risk of death during pregnancy. Risk of aortic dissection with an aortic root <40mm is 1% and approximately 10% if the diameter is greater than 40mm. If the aortic root diameter is >50mm elective aortic root repair should be done prior to pregnancy. During pregnancy goals of treatment are to decrease the pulsatile forces on the aorta with the use of a β-blocker. Aortic septal defect (ASD) is the most common congenital heart lesion and results in left to right shunt. Women usually do not have any symptoms and physical findings are often subtle. Pregnancy is well tolerated unless pulmonary hypertension develops. A small percentage of patients have arrhythmia most commonly atrial flutter or atrial fibrillation. If these develop they should be treated accordingly with antiarrhythmics. Williams Obstetrics 23rd Ed., Chapter 44 Cardiovascular Disease Creasy and Resnick 7th Ed, Chapter 52 Cardiac Disease

How well did you know this?
1
Not at all
2
3
4
5
Perfectly
11
Q
  1. A 27 year old G1P0 at 36 weeks dated by her last menstrual period consistent with an 8 week ultrasound presents for a growth ultrasound for size less than dates. Her ultrasound performed at 19 weeks showed normal anatomy. Today’s ultrasound shows the overall estimated fetal weight at 8th%ile, amniotic fluid volume of 16 cm, and normal umbilical artery dopplers. The patient wants to know how much weight the fetus should be gaining weekly at this point in her pregnancy. What do you tell her is the median weight gain per week?

A. 50 grams per week
B. 150 grams per week
C. 200 grams per week
D. 250 grams per week
E. 100 grams per week

A

C. Fetal growth restriction (FGR) affects less than 10% of infants born in developed countries and up to 30% of infants born in developing countries. Varying definitions of fetal growth restriction and the use of different growth curves contribute to the wide variation of FGR rates. A publication from 1982 reported outcomes of over 2.2 million live births in California from 1970 – 1976. Using the 50th percentile, growth curves were plotted for singletons and twin gestations. For singletons, the growth rate peaks at 250 grams at 33 weeks. For the patient in this question, at 36 weeks the median growth rate is approximately 200 grams per week. Of note, singleton and twin growth curves are nearly identical through 26 weeks.

How well did you know this?
1
Not at all
2
3
4
5
Perfectly
12
Q
  1. A 27 year old G1P0 at 36 weeks dated by her last menstrual period consistent with an 8 week ultrasound presents for a growth ultrasound for size less than dates. Her ultrasound performed at 19 weeks showed normal anatomy. Today’s ultrasound shows the overall estimated fetal weight at 8th%ile, amniotic fluid volume of 16 cm, and normal umbilical artery dopplers. The patient wants to know how much weight the fetus should be gaining weekly at this point in her pregnancy. What do you tell her is the median weight gain per week?

A. 50 grams per week
B. 150 grams per week
C. 200 grams per week
D. 250 grams per week
E. 100 grams per week

A

C. Fetal growth restriction (FGR) affects less than 10% of infants born in developed countries and up to 30% of infants born in developing countries. Varying definitions of fetal growth restriction and the use of different growth curves contribute to the wide variation of FGR rates. A publication from 1982 reported outcomes of over 2.2 million live births in California from 1970 – 1976. Using the 50th percentile, growth curves were plotted for singletons and twin gestations. For singletons, the growth rate peaks at 250 grams at 33 weeks. For the patient in this question, at 36 weeks the median growth rate is approximately 200 grams per week. Of note, singleton and twin growth curves are nearly identical through 26 weeks.

How well did you know this?
1
Not at all
2
3
4
5
Perfectly
13
Q
  1. Ms AT is an asian G1P0 female at 28 weeks gestation with a singleton fetus demonstrating findings of hydrops fetalis but no identifiable structural anomalies on detailed obstetric imaging and fetal echocardiography. No arrhythmia is identified, and the peak systolic velocity of the fetal middle cerebral artery measures 1.8 MoMs. Her blood type is O with Rh(D) positivity, and her antibody screen is negative. Kleihauer-Betke testing is negative. The maternal hemoglobin concentration is 10.5 gm/dL with an MCV of 73 fL. Amniocentesis reveals a normal karyotype with negative polymerase chain reaction studies for parvovirus, toxoplasmosis, and cytomegalovirus infection. What is the most likely cause of the hydrops fetalis?

A. Red cell alloimmunization
B. Fetal hemorrhage
C. Infection
D. Lysosomal storage disease
E. Fetal hemoglobinopathy

A

E. This case represents non-immune hydrops fetalis associated with fetal anemia (as denoted by the increased MCA peak systolic velocity > 1.5 MoMs). The work-up has been thorough and has identified a potential cause. Her negative antibody screen excludes maternal red cell alloimmunization as the inciting event. The negative Kleihauer-Betke screen excludes fetal hemorrhage. The amniocentesis results make fetal infection or aneuploidy unlikely causes. Metabolic disorders such as lysosomal storage diseases account for 1-2% of cases of NIHF and are rare. This woman’s Asian heritage, mild anemia, and low MCV (< 80 fL) indicate the potential for her to have alpha thalassemia minor. Alpha thalassemia is an autosomal recessive disorder common in the Southeast Asian population where it accounts for 28-55% of cases of NIHF. There are 4 functional alpha-globin genes responsible for the production of the alpha-globin chains of normal adult hemoglobin. Women with loss of two of the four alpha-globin genes are said to have alpha thalassemia minor, and Asian women tend to have the “cis” deletional form in which both alpha genes on one of the two chromosomes have been deleted (aa,–). If this woman’s partner also has alpha thalassemia minor, their offspring have a 25% chance of loss of all four alpha-globin genes (–,–) and having no functional alpha-globin production – Hemoglobin Barts, which presents with hydrops fetalis and is incompatible with extrauterine life. If the partner has alpha thalassemia minima with loss of only one of the four alleles, their offspring have a 25% of having Hemoglobin H disease with loss of 3 of the 4 alleles and hemolytic anemia and occasionally hydrops fetalis.

Society for Maternal-Fetal Medicine (SMFM), Norton ME, Chauhan SP, Dashe JS. Society for Maternal-Fetal Med

How well did you know this?
1
Not at all
2
3
4
5
Perfectly
14
Q
  1. A 27 year old F2P1001 at 32 weeks gestation comes in for a follow-up obstetric visit. Her health is complicated by diabetes and asthma. At her last visit, she had elected to further discuss antenatal Tdap vaccination next time. Today, she questions the benefits of receiving a Tdap vaccine. Which of the following is NOT a benefit of tdap vaccination?
    A. The Tdap vaccine has been given during pregnancy without reportable safety concerns
    B. The tdap vaccine is important to public health as a booster vaccine in order to minimize the circulating pool of available hosts given waning immunity
    C. Newborns do relatively well should they become infected with bordetella pertussis
    D. The tdap vaccine is ebenfical to her in reducing her change of getting pertussis in light of rising baseline incidence
    E. Antenatal tdap vaccine leverages vertical transmission of maternal antibodies
A

C. Pertussis (whooping cough) is an acute and prolonged infectious cough illness caused by Bordetella pertussis. The prevalence of pertussis in the United States has been increasing due to waning immunity in adults. Adults who develop whooping cough can pass it on to susceptible infants who are unvaccinated. Whooping cough can cause serious and occasionally life-threatening infections in babies, especially in the first six months of life. Infants do not get their first pertussis vaccine until they are 2 months of age, leaving a time period of vulnerability between birth and vaccination. In 2013 the Advisory Committee on Immunization Practices (ACIP) of the CDC issued guidelines recommending that women receive the Tdap vaccine in the third trimester of pregnancy regardless of when their last vaccine was given in order to maximize maternal antibody response and passive antibody transfer to the fetus. The optimal timing of vaccination is between 27-36 weeks gestation. No adverse reactions have been attributed to the Tdap vaccine. The most common side effects are local reactions such as redness and pain at the injection site. . ACOG Committee Opinion 566: Update on Immunization in Pregnancy: Tetanus, Diphtheria, and Pertussis Vaccination.

How well did you know this?
1
Not at all
2
3
4
5
Perfectly
15
Q

. What happens to apocrine and eccrine gland activity in pregnancy?
A. Decrease in apocrine gland activity, increase in eccrine gland activity
B. Decrease in apocrine gland activity, decrease in eccrine gland activity
C. Increase in apocrine gland activity, increase in eccrine gland activity
D. Increase in apocrine gland activity, decrease in eccrine gland activity

A

A. There are two main types of sebaceous glands. The apocrine glands empty into the hair follicles and are found primarily in the axilla and groin. The eccrine glands are found throughout the body in varying densities. It is believed that apocrine activity decreases during pregnancy, leading to an improvement in Fox-Fordyce disease and hidradenitis suppurativa. Conversely, eccrine activity increases in pregnancy leading to exacerbation of miliaria, hyperhidrosis, and dyshidrosis.

How well did you know this?
1
Not at all
2
3
4
5
Perfectly
16
Q
  1. Which of the following is NOT an essential element of appropriate blood pressure measurement?
    A. Use a sitting or semi-reclining position with back supported and arm at heart level
    B. Obtain correct size cuff: width of bladder 40% of circumference and encircles 80% of arm
    C. Patients feet should be flat, not dangling from examination table or bed and her legs uncrossed
    D. Consumption of nicotine or caffeine within 30 minutes of measurement should delay instituting antihypertensive therapies for severe range blood pressures as the blood pressure is likely falsely elevated and will likely soon return to normal
    E. Have the patient to sit quietly for five minutes prior to assessment
A

Answer:
D. One of the most important factors in the identification and management of hypertension in pregnancy is the standardized and appropriate measurement of blood pressure. The following details the essential aspects of measuring blood pressure in the obstetric patient.
Step 1: Prepare equipment.
1. Mercury sphygmomanometer is the gold standard; can use validated equivalent automated equipment
a. Check cuff for any defaults.
b. Obtain correct size cuff: width of bladder 40% of circumference and encircle 80% of arm.
Step 2: Prepare the patient.
1. Use a sitting or semi-reclining position with back supported and arm at heart level.
2. Have patient sit quietly for 5 minutes prior to measurement.
3. Bare upper arm of any restrictive clothing.
4. Patient’s feet should be flat, not dangling from examination table or bed, and her legs uncrossed.
5. Assess any recent (within the previous 30 minutes) consumption of caffeine or nicotine. If blood pressures are at the level that requires treatment, consumption of nicotine or caffeine should not lead to delays in instituting appropriate antihypertensive therapies.
Step 3: Take measurement.
1. Support patient’s arm at heart level, seated in semi-fowlers position.
2. For auscultatory measurement: use first audible sound (Kortokoff I) as systolic pressure and use disappearance of sound (Kortokoff V) as diastolic pressure.
3. Read to the nearest 2 mm Hg.
4. Instruct the patient not to talk.
5. At least one additional reading should be taken within 15 minutes.
6. Use the highest reading.
7. Do not reposition the patient to either side to obtain a lower BP. This will give you a false reading.
Step 4: Record measurement.
1. Document BP, patient position, and arm in which taken.
CMQCC Preeclampsia Toolkit. Accurate Blood Pressure Measurement. https://www.cmqcc.org/resource…Accessed May 27, 2021.
Peters RM. High blood pressure in pregnancy. Nurs Womens Health. 2008;12(5):410-422

How well did you know this?
1
Not at all
2
3
4
5
Perfectly
17
Q

. Which of the following correctly identifies changes in respiratory physiology in pregnancy?
A. Respiratory rate increases, tidal volume unchanged, total lung capacity decreases, PaO2 levels increase
B. Respiratory rate unchanged, tidal volume decreases, total lung capacity decreases, PaO2 levels increase
C. Respiratory rate unchanged, tidal volume increases, total lung capacity decreases, PaO2 levels increase
D. Respiratory rate decreases, tidal volume decreases, total lung capacity decreases, PaO2 levels decreases
E. Respiratory rate increases, tidal volume increases, total lung capacity increases, PaO2 levels increase

A

Definitions:
Respiratory Rate – number of breaths per minute
Vital capacity – maximal amount of air that can be forcibly expired after maximal inspiration (inspiratory capacity + Expiratory reserve volume)
Inspiratory capacity – Maximal amount of air that can be inspired from resting expiratory level (total volume + inspiratory reserve volume)
Tidal volume – Amount of air inspired and expired with a normal breath
Inspiratory reserve volume – maximal amount of care that can be inspired at the end of normal respiration
Functional residual capacity – amount of air in the lungs at resting expiratory level (expiratory reserve volume + residual volume)
Expiratory reserve volume – maximal amount of air that can be expired from resting expiratory level
Residual volume – Amount of air in the lungs after maximal expiration
Total lung capacity – total amount of air in lungs at maximal inspiration (vital capacity + residual volume)

C. During pregnancy, the diaphragm rises approximately 4 cm and the subcostal angle widens allowing the transverse diameter of the rib cage to increase by approximately 2 cm. This rise in the diaphragm leads to a decrease in the total lung capacity and functional residual capacity (includes expiratory reserve volume and residual volume which both decrease). Functional residual capacity decreases by approximately 20%. Tidal volume and minute ventilation increase significantly as pregnancy advances. The increases in tidal volume are approximately 30-50%, which is likely due to increased respiratory drive related to increases in progesterone. Although there is an increase in tidal volume, the respiratory rate remains unchanged during pregnancy. Pregnancy is a state of chronic mild hyperventilation which results in increased alveolar oxygen (PaO2) and decreased arterial carbon dioxide (PaCO2). The lower PaCO2 results in a chronic respiratory alkalosis. This is compensated through an increase in excretion of bicarbonate which maintains a relatively stable pH.

How well did you know this?
1
Not at all
2
3
4
5
Perfectly
18
Q
  1. Which of the following is the most common infectious cause of non-immune hydrops?
    A. CMV
    B. Parvovirus
    C. Toxoplasmosis
    D. Syphilis
    E. Rubella
A

Answer:
B. Parvovirus B19 infection is caused by a DNA virus that is transmitted primarily by respiratory droplets and infected blood products. Immunity increases progressively with age such that 50-60% of reproductive-age women have evidence of prior infection and immunity. The most common clinical manifestation of infection is erythema infectiousum (5th disease), which usually includes a low-grade fever, malaise, myalgias, arthralgias, and a “slapped cheek” facial rash. After acute parvovirus B19 infection during pregnancy, rates of maternal to fetal transmission range from 17-33%. Most cases resolve spontaneously, but an estimated 8-10% (and potentially up to 18-27%) of cases of non-immune hydrops fetalis are associated with parvovirus B19 infection. The virus is cytotoxic to erythroid precursors, and hydrops most often results from aplastic anemia, although hydrops can also be related to myocarditis or chronic fetal hepatitis. Severe effects are seen most frequently among fetuses when maternal infection occurs before 20 weeks of gestation.

Duff, P. (2014) Maternal and Fetal Infections In R.K. Creasy, R. Resnik, J.D. Iams, C.J. Lockwood, T.R. Moore and M.F. Greene. (Eds.) Creasy & Resnik’s Maternal-Fetal Medicine (7th ed., pp 838-9). Philadelphia, PA: Elsevier-Saunders.

ACOG Practice Bulletin no. 151: Cytomegalovirus, parvovirus b19, varicella zoster, and toxoplasmosis in pregnancy. Obstet Gynecol. 2015 Jun; 125(6) 1510-25.

How well did you know this?
1
Not at all
2
3
4
5
Perfectly
19
Q

what does PCWP measure

A

Pulmonary capillary wedge pressure (PCWP) is a measurement used to assess the left side of the heart and estimate the pressure within the left atrium. It indirectly reflects the pressure within the pulmonary veins and the left ventricle.

PCWP is typically measured using a pulmonary artery catheter (also known as a Swan-Ganz catheter) that is threaded through the right side of the heart and into the pulmonary artery. The balloon at the tip of the catheter is inflated and then wedged into a small branch of the pulmonary artery. This occludes the blood flow, allowing the measurement of pressure within the pulmonary capillaries.

PCWP is an important parameter in evaluating cardiac function and assessing the severity of heart failure. It is commonly used to estimate left ventricular end-diastolic pressure (LVEDP), which is a reflection of the filling pressure of the left ventricle during diastole. PCWP is also used to guide the management of certain cardiac conditions, such as determining optimal fluid status or assessing the effectiveness of interventions.

By measuring PCWP, healthcare professionals can gather information about the pressures within the left side of the heart and make informed decisions regarding treatment and management strategies for cardiac patients.

How well did you know this?
1
Not at all
2
3
4
5
Perfectly
20
Q
  1. Which of the following describe the changes that occur to cardiac physiology during pregnancy?

A. Increased cardiac output, decreased systemic vascular resistance, decreased maternal heart rate, increased pulmonary capillary wedge pressure
B. Decreased cardiac output, decreased systemic vascular resistance, increased maternal heart rate, increased pulmonary capillary wedge pressure
C. Increased cardiac output, increased systemic vascular resistance, unchanged maternal heart rate, unchanged pulmonary capillary wedge pressure
D. Increased cardiac output, decreased systemic vascular resistance, increased maternal heart rate, unchanged pulmonary capillary wedge pressure
E. Increased cardiac output, decreased systemic vascular resistance, decreased maternal heart rate, decreased pulmonary capillary wedge pressure

A

D. Cardiac output increases by approximately 30-50%, with approximately 50% of the increase occurring by 8 weeks of gestation. Increases in cardiac output are related to an increase in preload, which is due to the associated increase in blood volume; a decrease in afterload, which is due to a decrease in systemic vascular resistance; and an increase in maternal heart rate. Cardiac output appears to peak between 25-30 weeks of gestation. Systemic vascular resistance decreases in pregnancy, usually occurring early in gestation and nadirs by the mid-second trimester with increases occurring gradually in the third trimester. At term, the systemic vascular resistance remains 21% lower than pre-pregnancy values. Changes in vascular resistance are related to progesterone-mediated smooth muscle relaxation leading to vasodilation. The factors related to vasodilation are not completely understood but appear to be related to increased endothelial prostacyclin and increased nitric oxide production. Maternal heart rate increases by approximately 15-20 bpm during pregnancy and is the major contributor to increases in cardiac output towards the end of pregnancy. Plasma volume increases by approximately 10-15% in early pregnancy and continues to increase throughout pregnancy. Pulmonary vascular resistance decreases significantly during pregnancy; however, pulmonary capillary wedge pressure and central venous pressure do not change during pregnancy. Increases in pulmonary capillary wedge pressure increase the risk of pulmonary edema.

Williams Obstetrics 23rd ed., Chapter 5: Maternal Physiology
Gabbe’s Obstetrics: Normal and Problem Pregnancies 6th ed., Chapter 3: Maternal Physiology.

How well did you know this?
1
Not at all
2
3
4
5
Perfectly
21
Q
  1. Which of the following is the most abundant secretory hormone produced by the placenta?
    A. Prolactin
    B. Estriol
    C. hCS (human chorionic somatomammotropin)
    D. DHEAS
    E. Progesterone
A

C. The correct choice is hCS, human chorionic somatomammotropin. This is a peptide hormone similar to human growth hormone (hGH) and prolactin in structure. When it was first isolated, it was named human placental lactogen (hPL). Since then, it has been found to have minimal effect on lactation and therefore that designation is a misnomer. There are three genes coding for hCS is on the long arm of chromosome 17, and these are expressed in syncytiotrophoblast cells. The transcription rate of the hCS genes is constant throughout gestation, and as placental mass increases the rate of production increases accordingly. At term, the placenta produces approximately 1-4 grams of hCS per day, and the levels in the maternal serum range from 5-15 μg/mL. Despite its large quantities, the function of hCS is not well defined. Given its homology to hGH, it is hypothesized that it has functions in fetal growth and nutrition, however normally grown healthy infants have been born to women with low or absent levels of hCS.

How well did you know this?
1
Not at all
2
3
4
5
Perfectly
22
Q
  1. Which of the following is the most abundant secretory hormone produced by the placenta?
    A. Prolactin
    B. Estriol
    C. hCS (human chorionic somatomammotropin)
    D. DHEAS
    E. Progesterone
A

C. The correct choice is hCS, human chorionic somatomammotropin. This is a peptide hormone similar to human growth hormone (hGH) and prolactin in structure. When it was first isolated, it was named human placental lactogen (hPL). Since then, it has been found to have minimal effect on lactation and therefore that designation is a misnomer. There are three genes coding for hCS is on the long arm of chromosome 17, and these are expressed in syncytiotrophoblast cells. The transcription rate of the hCS genes is constant throughout gestation, and as placental mass increases the rate of production increases accordingly. At term, the placenta produces approximately 1-4 grams of hCS per day, and the levels in the maternal serum range from 5-15 μg/mL. Despite its large quantities, the function of hCS is not well defined. Given its homology to hGH, it is hypothesized that it has functions in fetal growth and nutrition, however normally grown healthy infants have been born to women with low or absent levels of hCS.

How well did you know this?
1
Not at all
2
3
4
5
Perfectly
23
Q

. A 22 year old G1P0 at 35 weeks gestation with a history of childhood asthma is seen in the clinic. Which blood gas would be most consistent with a healthy pregnancy at sea level? Answers given as arterial pH/Arterial PO2 (mmHg)/Arterial PCO2 (mmHg)/Sodium bicarbonate (mEq/L)
A. 7.4/102/28/20
B. 7.3/78/23/20
C. 7.5/100/23/16
D. 7.4/100/45/26

A

A. Due to an increase in tidal volume and decrease in functional residual capacity, there is a relative hyperventilation in pregnancy, which results in a compensated respiratory alkalosis. The pH remains around the non-pregnant level (7.4-7.48). The arterial PO2 is moderately increased due to greater alveolar ventilation (101-104 mm Hg). The PCO2 is decreased (26.9-32.5 mm Hg) due to a combination of the hyperventilation effects of progesterone and the increased production of CO2 related to the increased metabolic rate. To compensate for decreased CO2, there is an equivalent increase in the renal excretion of bicarbonate to maintain an unchanged pH (bicarbonate range in pregnancy 18-26 mEq/L).

How well did you know this?
1
Not at all
2
3
4
5
Perfectly
24
Q
  1. A 35 year old G3P2 female at 28 weeks of gestation with a history of moderate persistent asthma was recently hospitalized for an asthma exacerbation and is being sent home on a medium-dose inhaled corticosteroid and an albuterol rescue inhaler. She plans on using a peak expiratory flow rate meter (PEFR). How should she be counseled regarding response to the inhaler?

A. A good response is defined as PEFR 60% of expected
B. A good response is if her PEFR is 224 L/min
C. An incomplete response is defined as PEFR <50% and she should repeat her albuterol treatment at 20-minute interval s up to 2 more times
D. An incomplete response is defined as PEFR 80% or less of expected, and she should use albuterol treatment at 20 minute intervals up to 2 more times and reassess PEFR
E. A poor response is defined as PEFR 70% of expected adns she should repeat her albuterol treatment and obtain emergency care

A

D. The correct answer is: an incomplete response is defined as PEFR 80% or less of expected and she should use albuterol treatment at 20-minute intervals up to 2 more times and reassess her PEFR. PEFR is the forced expiratory volume in 1 minute. It can be used in pregnancy to help determine the severity of asthma in patients and the appropriate response to treatment. Although the PEFR varies with position (standing > sitting > supine) the value is stable throughout gestation (>320 standing, >310 sitting, >300 supine). A patient’s asthma can be monitored by % of the predicted best.
In women with intermittent and mild persistent asthma, their PEFR is >80%. When the PEFR decreases to 60-80% of the predicted best, this would be considered not controlled moderate persistent asthma. When asthma is very poorly controlled and the PEFR is <60%, this is considered severe persistent asthma.
Home management of acute asthma exacerbation has been outlined by the National Asthma Education and prevention program. An albuterol metered-dose inhaler (MDI) at a dose of 2-4 puffs is the initial line of treatment. A GOOD response is if PEFR is > 80% and there is no wheezing/shortness of breath and normal fetal movement. With a GOOD response, the MDI can be repeated every 3-4 hours as needed. An INCOMPLETE response is PEFR of 50-80% OR if wheezing/ shortness of breath persists. With an INCOMPLETE response, the MDI should be repeated at the same dose at 20-minute intervals up to 2 more times. If the repeat PEFR is still 50-80% of predicted or if decreased fetal movement, contact the caregiver or seek emergency care. A POOR response is if PEFR is <50% of predicted OR severe wheezing and shortness of breath or decreased fetal movement. The MDI should be repeated and emergency care should be obtained. Reference ranges for % of predicted PEFR are similar for hospitalized patients with a GOOD response being > 70%, INCOMPLETE response 50-70%, and POOR response < 50%.

How well did you know this?
1
Not at all
2
3
4
5
Perfectly
25
Q
  1. A 25 year old G1P0 female at 18 weeks gestation presents to you fro consultation regarding her history of SLE. She has a history of high titers of anti-Ro/SSA antibodies. SHe had an ultrasound today demonstrating a normal fetal heart rate with atrio-ventricular synchrony. How would you counsel her regarding potential for her fetus developing congenital heart block?

A. The risk of her fetus developing congenital complete heart block (CCHB) is small, but when it occurs CCHB associated with maternal anti-Ro/SSA antibodies is irreversible and carries a significant morbidity and mortality risk for the affected offspring.
B. Monitoring with weekly fetal echocardiograms to assess PR intervals is recommended, because a prolonged PR interval precedes more advanced heart block.
C. Anti-Ro/SSA antibodies do not cross the placenta at 18 weeks, so her fetus is not at risk for congenital heart block.
D. In this nullipara with SLE, hydroxychloroquine is recommended because it is proven to significantly reduce the risk of fetal heart block in any woman with anti-Ro/SSA antibodies.

A

A. Anti-Ro/SSA and anti-La/SSB antibodies cross the placenta and enter the fetal circulation to potentially induce fetal injury. Congenital complete heart block (CCHB) is one manifestation and most commonly occurs between 16 and 24 weeks of gestation. CCHB is irreversible due to fetal cardiac conduction system damage that occurs in utero. Other findings of neonatal lupus syndrome (NLS) include rash, hepatitis, hemolytic anemia, and thrombocytopenia, all of which are transient. NLS occurs in women with anti-Ro/SSA and/or anti-La/SSB, even if the mother is clinically well. In the absence of having previously delivered an infant with NLS, women with anti-Ro/SSA have a 2-3% risk of having a liveborn child with CCHB. However, among women with a previously affected child born with CCHB, the recurrence risk for heart block is 17% to 20%. The PRIDE study assessed the utility of monitoring with serial fetal echocardiograms performed from 16 to 26 weeks gestation to assess the PR interval. They found that prolongation of the PR interval was uncommon, and could not reliably be shown to precede CCHB. CCHB and cardiomyopathy were noted to occur within 1 week of a normal echocardiogram, suggesting that serial monitoring with fetal echocardiograms may not helpful for identifying which fetuses will develop CCHB, especially in women who have not previously had a fetus with CCHB.

No pharmacologic therapy has been found to reverse CCHB in an affected fetus; however, several therapies have been investigated in treating second-degree heart block, cardiomyopathy, and hydrops. Fluorinated steroids may be beneficial in treating second-degree heart block, although there is potential for major maternal and fetal side effects, so further study is warranted. In CCHB, beta-agonists have been used to increase the fetal ventricular rate, though they are associated with significant maternal side effects and their effect on reducing fetal/neonatal mortality remains unproven. Theoretically, plasmapheresis may aid in the prevention and treatment of fetal congenital heart block by lowering levels of anti-Ro/SSA and anti-La/SSB antibodies, but only case reports have been published. IVIG has shown promise in treating fetal cardiomyopathy but has not been shown to reduce the recurrence risk in women with a previously affected child. Retrospective studies suggest that hydroxychloroquine may decrease the recurrence risk of fetal congenital heart block in women with a previously affected child, and this drug has been used safely and regularly in pregnancy for other indications. A clinical trial is currently underway to further investigate the utility of hydroxychloroquine in this population of women.

How well did you know this?
1
Not at all
2
3
4
5
Perfectly
26
Q
  1. A 36 year old hispanic multiparous woman was diagnosed with peripartum cardiomyopathy three weeks following her most recent delivery. Which of the following is the most important factor for counseling regarding future pregnancies?

A. Ejection fraction at time of diagnosis
B. Severity of symptoms at time of presentation
C. Ejection fraction 12 months following diagnosis
D. Interval between delivery and presentation with symptoms

A

C. Peripartum cardiomyopathy or idiopathic cardiomyopathy in pregnancy is a diagnosis of exclusion following evaluation for other causes of heart failure. The incidence varies widely and is reported in the US as 1 in 3000 to 4000. Diagnostic criteria includes:
1. Development of heart failure during the last month of pregnancy or within 5 months postpartum
2. Absence of other identifiable causes of heart failure
3. Absence of recognizable heart disease prior to the last month of pregnancy
4. Left ventricular systolic dysfunction demonstrated by echocardiographic findings of decreased ejection fraction or shortening fraction
Women typically present with symptoms of heart failure including dyspnea, orthopnea, cough, palpitations, and chest pain. Findings on imaging studies are significant for cardiomegaly. Echocardiograph findings demonstrate an ejection fraction (EF) <45% or a shortening fraction <30%. Management consists of treating heart failure. Diuretics are used to reduce preload. Afterload reduction is achieved with use of a vasodilator. Digoxin is used if arrhythmias are identified. In women with severe cardiac dysfunction, anticoagulation may be recommended. Approximately 20% of patients have significant decline following diagnosis and require heart transplant. Partial recovery is seen in 30–50% of women, however, some degree of dysfunction remains. The remaining 30–50% of patients have significant improvement. Women who recover to a normal cardiac function and desire another pregnancy have a 20% risk of recurrence and low mortality rate. In contrast, women who do not return to normal cardiac function and undergo another pregnancy have a significant risk of recurrence and mortality. If the EF at diagnosis is <25%, 1 study found that at least 50% of these patients are subsequently placed on the cardiac transplant list. However, among patients whose EF was > 25% at diagnosis, none were placed on the cardiac transplant list during the next 3.4 ± 1.9 years. So although answer choices A and B do represent important prognostic factors for women with peripartum cardiomyopathy, the most important factor for future pregnancy counseling is whether or not the patient returned to normal cardiac function

Williams Obstetrics 23rd ed., Chapter 44: Cardiovascular Disease.

Creasy and Resnick’s Maternal-Fetal Medicine: Principles and Practice 7th ed., Chapter 52: Cardiac Disease.

How well did you know this?
1
Not at all
2
3
4
5
Perfectly
27
Q

A 26 year old G1P0 presents at 38 weeks in spontaneous labor and delivers precipitously. After delivery an active herpes simplex virus (HSV) lesion is noted on her labia. She has a history of genital HSV, but has been symptom free for several years. What is the best estimate for the risk of neonatal HSV infection due to this exposure?
A. <1%
B. 3%
C. 15%
D. 30-60%
E. >70%

A

B. Herpes simplex virus (HSV) is a common sexually transmitted infection that is acquired primarily through skin-to-skin contact, and then can reactivate secondarily throughout one’s lifetime. In 1 large study, 26% of women had serologic evidence of HSV-2 infection. Although HSV usually is asymptomatic, or only results in minor symptoms for the mother, it can occasionally present with a severe infection, including febrile illness and severe pain, particularly during a primary infection. HSV can be transmitted to the newborn and can have devastating effects. The mortality for neonatal HSV infection is 4% for central nervous system disease and 30% for disseminated disease. Additionally, approximately 20% of survivors have long-term neurologic sequelae. Although there are reports of transmission in utero, neonatal HSV is nearly always acquired during the intrapartum period through exposure to the virus in the genital tract. For women with a primary infection at the time of delivery, the risk of vertical transmission to the neonate is 30–60%. However, for women with an active recurrent HSV lesion, the risk is only 3%. For women with a history of HSV but no lesions at the time of delivery, the risk of vertical transmission is estimated to be 1 in 5,000. Cesarean delivery has been shown to significantly reduce the risk of vertical transmission, but it does not eliminate the risk. It is recommended in the setting of an active HSV lesion, whether primary or recurrent. It is also recommended for women with a history of HSV and prodromal symptoms, such as vulvar pain and burning. However, cesarean delivery is not recommended for women with a history of HSV and no active lesions at delivery. To reduce the risk of active HSV at the time of delivery, women with a history of HSV should be offered suppressive antiviral therapy at or beyond 36 weeks of gestation.

28
Q

An otherwise healthy G1P0100 whose prior pregnancy ended in unexplianed fetal death at 22 weeks gestation has been found to be repeatedly positive for lupus antiocoagulatn (LAC) on 3 occasions over the course of several months. She has heard that if she takes a heparin compound and low dose aspirin in her next pregnancy, her pregnancy likely will be successful. Regarding this expectation and other issues related to her current status, which fo the following is the best counseling for this patient?

A. She does not have antiphospholipid syndrome (APS) because she has not had a thrombotic event or recurrent miscarriage. Thus, the use of a heparin agent during her next pregnancy would be of no medical benefit to her or to her fetus.
B. In addition to fetal death, she is at increased risk for her fetus developing completed congenital heart block because she probably also has systemic lupus erythematosus. Fetal heart block would be best prevented by the use of intravenous immunoglobulin.
C. She should be treated in a next pregnancy with a heparin compound and low dose aspirin. Even with treatment, though, her being repeatedly positive for lupus anticoagulant portends an approximately 35% to 40% risk of an adverse pregnancy outcome.
D. She has APS, and if she has treatment during pregnancy with full dose (therapeutic) UFH or LMWH, her likelihood of a successful pregnancy delivered at or near term is at least 80%.\

A

C. The fact that this patient has an otherwise unexplained fetal death at 22 weeks and a repeatedly positive test for LAC indicates that she has APS. The other obstetric clinical features of APS are otherwise unexplained recurrent early miscarriage and delivery prior to 34 weeks for gestational hypertensive disease or placental insufficiency. To make the diagnosis, a patient must have at least 1 clinical feature (either a thrombotic event or an obstetric feature) and must be positive for either lupus anticoagulant or medium-to-high titer anticardiolipin (aCL) or anti-β2-glycoprotein I (aβ2-GP-I) antibodies (immunoglobulin G [Ig G] or immunoglobulin M [IgM] istoype) on at least 2 occasions, 12 weeks apart. Patients who test positive for LAC, and particularly those positive for LAC, aCL, and aβ2-GP-I antibodies (so called “triple positive”), appear to be at the greatest risk for adverse clinical events. The prospective observational PROMISSE study found that 39% of women with LAC had either a fetal death, an early delivery for gestational hypertensive disease, a small for gestational age birth weight, or neonatal death prior to discharge despite heparin treatment during the observed pregnancy. Pregnancy outcomes were far better in LAC-negative patients, and multivariate analysis found that aCL and aβ2-GP-I levels were not associated with the adverse outcomes studied. Thus, women with APS and LAC should be counseled that heparin treatment is imperfect with regard to its potential to improve APS pregnancy outcomes. There is no proven benefit to therapeutic versus thromboprophylactic doses of a heparinoid to improve fetal outcome in APS pregnancy, and most experts would recommend only thromboprophylactic doses for a patient like the one presented above.

Heparin treatment to prevent 2nd or 3rd trimester complications of APS has never been studied in a properly designed treatment trial. However, experts believe that such a trial is exceedingly unlikely, in part because of legitimate concerns about maternal thrombosis in APS pregnancy. Although an exact risk for maternal thrombosis in APS pregnancy is unknown, most experts believe the risk is sufficient to warrant thromboprophylaxis. Women with a diagnosis of APS and prior thrombosis are generally on long-term anticoagulation and are advised to use therapeutic doses of a heparin agent if they become pregnant. Interested investigators are currently exploring other treatments (in addition to heparin to prevent maternal thrombosis) that might improve fetal outcomes in APS pregnancy.

29
Q

. A 25 year old G2P1001 at 22 weeks presents to labor & delivery where she complains of abdominal pain, nausea, and constipation for the past 3 days. On physical examination she has no palpable abdominal masses and bowel sounds are present and active. Her lungs are clear to auscultation bilaterally. She has a heart rate of 69 beats/min with an irregular rhythm. An electrocardiogram demonstrates a shortened QT (corrected) gastroenterology consult is obtained and an upper gastrointestinal (GI) endoscopy reveals multiple 1 cm diameter shallow ulcerations of the gastric antrum. Which of the following laboratory test findings is most likely to be diagnostic int this woman?

A. Elevated thyroid peroxidase antibody
B. Elevated total serum thyroxine
C. Elevated morning plasma cortisol
D. Elevated serum calcium
E. Elevated upright plasma renin activity

A

D. The patient most likely has a parathyroid adenoma secreting excessive parathyroid hormone, which results in an increase in serum calcium and decreased serum phosphorus. The hypercalcemia leads to increased gastrin production and peptic ulcer disease. Hypercalcemia produces cardiac arrhythmias (or asystole). A positive thyroid peroxidase antibody suggests an autoimmune process such as Hashimoto thyroiditis. Graves disease can be associated with a high thyroxine level. Pituitary adenomas secreting adrenocorticotropic hormone (ACTH) to produce adrenal hyperplasia, or adrenal neoplasms, can cause secretion of excessive cortisol. Hyperreninemia may be seen with some forms of renovascular hypertension and with hyperaldosteronism with hypokalemia.

30
Q

. A pregnant woman presents for care with fever and myalgias. She reports she is concerned about listeriosis because she ate fruit from a company whose products were listed as contaminated by a recent centers for disease control and prevention (CDC) report. The most appropriate test to confirm listeriosis in this patient would be:

A. Targeted blood culture
B. Serum listeria-specific antibody testing
C. Stool Culture
D. Serum white blood cell count
E. No testing indicated, provide empiric antibiotics

A

A. Listeriosis is a disease caused by a bacterium, Listeria monocytogenes. This bacterium is present in certain contaminated foods. Multiple categories of foods have been implicated in outbreaks of listeriosis, including deli meats, unpasteurized dairy products, soft cheeses, raw vegetables and fruits, and smoked seafood. The most common disease presentation is fever and flu-like symptoms. In some cases there can be preceding diarrhea; rarely there can be serious complications such as cranial nerve deficits, endocarditis, or brain abscess. These severe cases have been reported in immunocompromised patients. The reported incidence of listeriosis in pregnancy is 12 per 100,000. Although listeriosis in pregnancy is rare, the incidence in pregnancy is about 13 times higher than the general population and the disease process can lead to obstetric complications such as miscarriage, stillbirth, preterm labor, and low birth weight as well as serious neonatal morbidities and mortality. Diagnosis requires targeted blood culture for L. monocytogenes; standard culture media are not adequate to detect this bacterium. Serum antibody testing is not available commercially. Stool culture is not an appropriate choice due to the existence of asymptomatic carriers; a culture from blood or amniotic fluid would yield the appropriate result. Because obtaining a blood culture carries less risk to the pregnancy than amniotic fluid culture, it is the most appropriate first step. A serum white blood cell count is not sufficiently specific for the diagnosis. In Committee Opinion 615, the American College of Obstetricians and Gynecologists recommends: Exposed pregnant women with a fever higher than 38.1 degrees Celsius and signs and symptoms consistent with listeriosis for whom no other cause of illness is known: empiric antibiotics (IV ampicillin) and targeted blood culture for diagnosis. Asymptomatic pregnant women who report consumption of a product that was recalled or implicated during an outbreak of listeria contamination: no testing, including blood and stool cultures, or treatment is indicated. A pregnant woman who ate a product that was recalled because of listeria contamination and who is afebrile but has signs and symptoms consistent with a minor gastrointestinal or flu-like illness can be managed expectantly.

31
Q

A 33 year old G1P0 female at 38 weeks presents after spontaneous rupture of membranes, and on examination her fetus is breech. Her medical history is significant for hyperthyroidism, well controlled medication, anxiety that she manages with behavioral techniques, and childhood asthma. She is otherwise healthy. You perform an uncomplicated cesarean section. Six hours after delivery, you are paged because she has a fever of 102 degrees F, heart rate of 145 beats/minute, and diarrhea. The nurse also airports that the patient is agitated. What is the most likely diagnosis?

A. Atelectasis
B. Viral gastroenteritis
C. Pulmonary embolus
D. Acute anxiety exacerbation
E. Thyroid storm

A

E. The correct answer is thyroid storm. Thyroid storm is a rare medical emergency, but carries a high risk of heart failure and mortality. It is a hypermetabolic state seen in women with hyperthyroidism. Thionamide therapy, even of short duration, is generally effective in preventing thyroid storm. Diagnosis is based on a constellation of symptoms including fever, tachycardia, altered mental status (typically restlessness, nervousness, or confusion), vomiting, diarrhea, and cardiac arrhythmias. The diagnosis is confirmed with biochemical evidence of hyperthyroidism (elevation of free T4 and/or T3 and suppression of thyroid-stimulating hormone [TSH]); however, a clinical diagnosis must be established and treatment initiated well before confirmatory thyroid function tests are available. There is often an inciting event such as labor, delivery, infection, or surgery.
Treatment should be initiated and consultation with critical care specialists sought. The patient needs close monitoring in an intensive care unit (ICU) setting. Medications that treat thyroid storm are standardized and include propylthiouracil (PTU), iodide, glucocorticoids such as dexamethasone, and beta blockers. In addition to the prompt initiation of pharmacotherapy, general supportive measures should be initiated including cooling blankets, intravenous fluids, oxygen, and electrocardiographic monitoring. Recognition and treatment of any precipitating factors is critical.
Creasy and Resnick, Maternal-Fetal Medicine, Chapter 47: Thyroid Disease and Pregnancy.

32
Q

For which of the following molecules is the net excretion greater in healthy pregnant women compared with nonpregnant women with healthy renal function?

A. Potassium
B. Uric acid
C. Sodium
D. Glucose
E. Calcium

A

D. The renal tubules undergo many changes in function due to the influence of various circulating hormones in pregnancy. Sodium homeostasis is regulated by competing sets of factors. The combination of increased glomerular filtration rate and the actions of progesterone, prostaglandins, and atrial natriuretic factor favor excretion of sodium into the urine. However, the opposing action of aldosterone, estrogen, deoxycorticosterone, and the influences of postural changes (supine and upright posture in particular) promote sodium reabsorption so that the overall balance is in favor of sodium retention, approximately 950 mg cumulative over the course of the pregnancy.
A similar process occurs with potassium homeostasis. Although the increased plasma aldosterone level should promote increased potassium excretion, there is enough reabsorption stimulated by progesterone at the proximal tubule such that there is a net retention of 300–350 mEq. Urinary calcium excretion increases during pregnancy, however the absorption of calcium at the intestine increases so that serum levels of unbound calcium remain neutral. Due to a decrease in plasma albumin, the bound fraction of calcium does decrease, leading to an overall decrease in total calcium levels. Glucose excretion in the nonpregnant patient is approximately 100 mg/day. In pregnancy, this increases 10- to 100-fold, because of impaired reabsorption at the collecting tubule and loop of Henle. Therefore, glycosuria cannot be used reliably in pregnancy to monitor diabetic patients. Uric acid levels decrease in the plasma by 25%, with a nadir of 2–3mg/dL at 24 weeks, then increase again toward nonpregnant levels by term. Certain conditions associated with volume contraction such as pre-eclampsia might be associated with a decrease in uric acid clearance and increase in plasma levels.
Creasy & Resnick. 7th Edition, Ch 7, Pgs 98-99.

33
Q

Which of the following is a prognostic indicator of poor outcome in septic shock?~~~
A. Delay in initial diagnosis
B. Increased oxygen extraction
C. Low serum lactate level (<4 mmol/L)
D. Increased cardiac output

A

A. Delay in diagnosing septic shock and therefore initiation of therapy is an important predicator of poor outcome in septic shock. Other factors that predict poor outcomes include, depressed cardiac output, high serum lactate (> 4 mmol/L) and reduced oxygen extraction. The management and goal of therapy of sepsis in pregnancy is the same for the nonpregnant patient. However, the etiologies of sepsis may be unique to pregnancy and the fetus needs to be considered.
Obstet Gynecol. 2012 Sept;120(3):689-706.

34
Q

28 year old multigravida woman is brought to the emergency department via emergency medical services, where she was found unresponsive. On the scene, there was report of copious quantities of black vomit present. The paramedics report that the woman is a diabetic and gives herself injections. On physical examination, her vital signs are blood pressure 101/72 mmHg, heart rate 123 beats/minute, respiratory rate 32 breaths/minute, and oxygen saturation is 100% on room air. Her lungs are clear to auscultation bilaterally, although her respiratory pattern is that of rapid and deep breathing. Her abdominal examination is unremarkable. Her initial laboratory results reveal a blood glucose value of 288 mg/dL, serum sodium of 127, potassium 5.3, chloride of 87, and serum creatinine of 1.5 mg/dL. Volume resuscitation and insulin treatment are initiated. As treatment is in progress, the potassium level falls to 3.3. What is the MOST likely contributing factor to her hypokalemia?
A. Continuous insulin infusion
B. Rapid decline of serum osmolarity
C. Use of hypotonic intravenous fluid
D. Intracellular shifting of potassium ions
E. Rapid infusion of sodium bicarbonate treatment

A

D. Diabetic ketoacidosis (DKA) during pregnancy is a serious complication of diabetes resulting in increased risks for the mother and fetus. Although maternal morbidity and risk of fetal loss has decreased over the years, DKA is a medical emergency during pregnancy. DKA occurs more frequently in mothers with type 1 diabetes, but can occur in those with type 2 diabetes.
There are a number of precipitating risk factors that can predispose a pregnant woman to DKA including but not limited to: infection (urinary and pulmonary are most common), previously undiagnosed diabetes, poor compliance with taking insulin as prescribed, insulin pump failure, stress, dehydration, and decreased caloric intake from persistent nausea and vomiting.
Treatment involves intravenous (IV) fluids to correct dehydration, insulin to suppress the production of ketone bodies, treatment for any underlying causes such as infections, and close observation to prevent and identify complications.The use of continuous infusion of insulin allows frequent adjustments of insulin to correct hyperglycemia. IV regular insulin bolus (0.1 U/kg body weight), followed by continuous infusion of 0.1–0.2 U/kg/hr to gradually correct the patient’s hyperglycemia is indicated and attempts should be made to not correct the patient’s plasma glucose level too rapidly. If the glucose levels drop too quickly, excessive movement of water into cells may occur from the rapid drop in serum osmolarity. The use of hypotonic IV fluid instead of isotonic saline for the initial volume replacement may result in a rapid decline of serum osmolarity that may increase the patient’s risk for cerebral edema. Use of bicarbonate to increase pH is controversial. The rapid infusion of sodium bicarbonate can cause paradoxical central nervous system acidosis. One proposed algorithm for management is included below but there are several possible management plans. Laboratory values on diagnosis generally include metabolic acidosis with an anion gap [serum anion gap = serum sodium – (serum chloride + bicarbonate)], hyperglycemia (usually marked although DKA has been found in pregnancy even with mild elevation in serum glucose), ketones in serum and urine, elevated creatinine, and low serum bicarbonate. Although most patients will initially have normal or even elevated serum potassium, there is an overall potassium deficit secondary mainly to renal losses. As fluid status and glucose normalize with treatment, potassium ions will move into the intracellular space, unmasking the overall hypokalemia. Severe hypokalemia can result, especially in patients who initially present with normal or low serum potassium levels. Careful monitoring and replacement of potassium are essential in the treatment of DKA. If presenting potassium levels are less than 3.3, insulin therapy should be delayed until potassium replacement has been started because insulin therapy will worsen the hypokalemia. Proposed management of pregnant patient with DKA: Fetal assessment: Lateral uterine displacement, oxygen therapy, possible fetal monitoring.

Maternal Assessment: Detailed history and physical examination, oxygen saturation monitoring, rule out infection and search for underlying cause (eg, urine culture, chest radiograph, and so on)
Volume replacement: transurethral catheter, maintain urine output at >50 mL/hour, initial IV fluids with normal saline (NS) at 1L/hr x2 hr and then convert to ½ NS at 250 mL/hr. When serum glucose is less than 200–250 mg/dL, convert to dextrose 5% (D5) 1/2 NS at 250 mL/hr and continue for 24–48 hr.
Insulin therapy (regular): IV bolus with 0.1–0.2 U/kg or 10–15 units. Follow with insulin infusion of 0.1 U/kg/hr to decrease serum glucose by 50–75 mg/dL/hr. If serum glucose does not decrease at least 50 mg/dL in the first hour, double the rate of infusion. When serum glucose is less than 200 mg/dL, decrease rate to 0.05 U/kg/hr. Serum glucose goal: 150–200 mg/dL. May convert to subcutaneous insulin once the patient is stable and tolerating oral intake (usually 24–48h after presentation).
Potassium management: Based on initial potassium level and normal renal output. If serum potassium is: Less than 3.3 mEq/L then hold insulin infusion and replace potassium. Greater than 5.3 mEq/L, repeat every 2 hr until is less than 5.3 mEq/L. Between 3.3 and 5.3 mEq/L, add 20–30 mEq to each liter of replacement fluids to maintain potassium in the range of 4–5 mEq/L.
Bicarbonate management: No NaHCO3 if maternal pH greater than 7. If less than or equal to 7, give 100 mmol of NaHCO3 in 500 mL 1/2 NS with 20 mEq of potassium over 2 hr and repeat every 2 hr until greater than 7. Bicarbonate is not the first line therapy for correcting acidemia, volume resuscitation and insulin are first line. Only if those therapies do not succeed in correcting the acidemia should bicarbonate be considered.

Creasy and Resnick, Chapter 61 Other Endocrine Disorders of Pregnancy.

American College of Obstetricians & Gynecologists Practice Bulletin Number 60, Pregestational Diabetes Mellitus. Reaffirmed 2012.

Belfort M. Critical Care Obstetrics, 5th ed. Chapter 33 Endocrine Emergencies

.

35
Q
  1. A previously healthy 23-year old G1P0 presents to the hospital with new onset of severe headache and right upper quadrant pain. Her blood pressure is 190/115 mmHg. Which statement below depicts the most accurate initial dosage, infusion rate, frequency, and maximal 24 hour cumulative dosage for the antihypertensive agent in question?

A. Hydralazine 10 mg IV push over 2 minutes repeated every 10 minutes to a maximum of 30 mg/24 hours
B. Hydralazine 10 mg IV push repeated every 10 minutes to a maximum of 30 mg/24 hours
C. Hydralazine 20 mg IV push repeatedly every 10 minutes to a maximum of of 40 mg/24 hours
D. Labetalol 20 mg IV push with a doubling of the dose every 20 minutes to a maximum of 300 mg/24 hours
E. Labetalol 20 mg IV push over 2 minutes with a doubling of the dose every 10 minutes to a maximum of 300 mg/24 hours

A

E. The 2 most common first-line intravenous (IV) antihypertensive agents used in the obstetric patient are labetalol and hydralazine. Hydralazine is an arteriolar vasodilator that reduces blood pressure (BP) but may cause tachycardia. Labetalol is a combined alpha and beta-blocking agent that reduces BP by dilating arterioles and decreasing heart rate and cardiac output. Some have advocated using labetalol as a first-line agent for all patients who do not have contraindications for beta-blockers: asthma, acute cocaine or methamphetamine exposure, or bradycardia, (heart rate [HR] <60 beats/min). Initial therapy should consist of labetalol 20 mg or hydralazine 5–10 mg IV over 2 minutes. Hydralazine begins to have an effect within 5–20 minutes with its maximum effect occurring at 15–-30 minutes. Labetalol’s onset is within 2–5 minutes and has its maximum effect after 5 minutes. Both should be administered intravenously over 2 minutes. The accepted algorithm for labetalol use is to start with 20 mg IV over 2 minutes, then 40 mg IV administered 10 minutes later if the blood pressure remains greater than 160/110 mmHg. Two more doses of 80 mg IV can be administered every 10 minutes if the BP remains severe (>160/110 mmHg) to a maximal cumulative dose of 220 mg/24 hours. The recommended dosing strategy for hydralazine is to start with 5–10 mg IV over 2 minutes followed by an additional 10 mg dose IV over 2 minutes if severe hypertension persists 20 minutes after the initial dose. This 10 mg dose can be repeated 20 minutes later for persistently severe hypertension for a maximal cumulative dose of 25 mg/24 hours.

ACOG Maternity Safety Bundle for Severe Hypertension in Pregnancy.

36
Q

. Which of the following answers is TRUE concerning the perinatal/vertical transmission of cytomegalovirus (CMV)?
A. CMV is the least common congenital infection
B. Vertical transmission rates after recurrent or reactivation of CMV infection are about the same as the rates with a primary CMV infection
C. Vertical transmission to the fetus is higher when maternal infection occurs in the third trimester
D. More serious sequelae occur with CMV infection in the second trimester

A

C. CMV is the MOST common congenital infection occurring in 0.2 to 2.2% of neonates. Vertical transmission rates with primary CMV infection are 30–70% whereas recurrent infection transmission rate is just 0.15–2%. Vertical transmission to the fetus is highest in the third trimester (40–72%). The transmission rate is 30% in the first trimester and 34–38% in the second trimester. Although the first trimester transmission rates are lower than third trimester rates, the more serious CMV sequelae are noted with first trimester maternal CMV infection (up to 25% of affected pregnancies in the first trimester).

ACOG Practice Bulletin #151, June 2015. CMV, Parvovirus B19, Varicella Zoster, and Toxoplasmosis in Pregnancy.

37
Q
  1. A 26 year old G1P0 female at 33 weeks presents to triage with 5 days of nausea, vomiting, and abdominal pain. Her pregnancy is complicated by type I diabetes. The fetal heart rate tracing shows the heart rate in the 150s (beats/min) with minimal variability and late decelerations. Which of the following laboratory results is most consistent with the above presentation?
    A. Anion gap 11
    B. Maternal arterial pH 7.35
    C. Blood glucose 180 mg/dL
    D. Negative ketonuria
A

C. Diabetic ketoacidosis (DKA) is still a serious cause of morbidity and mortality, including fetal loss, despite improved recognition and treatment. Although focus should remain on prevention, recognition of such events is extremely important and can help prevent significant maternal and fetal complications. Pregnant women are predisposed to DKA from decrease in caloric intake from pregnancy-associated nausea, accelerated starvation, dehydration, and an increase in insulin antagonists. However, more common precipitating events include infection and noncompliance. In the workup for DKA, physicians should always check for evidence of infection.

Pregnant women can have DKA with blood glucose under 200mg/dL. To optimize outcomes, suspicion must be heightened and the diagnosis made quickly. Women with DKA present with ketonuria and acidosis (pH <7.30). The anion gap will be increased to >12 and serum bicarbonate will be decreased. There is a significant loss of total body sodium and potassium. As the acidosis improves, potassium ions shift intracellularly. It is therefore important to follow potassium levels closely and replace as needed. Initiation of insulin should be performed to reverse metabolic abnormalities. Focus should be on aggressive fluid replacement to correct volume deficit and improve tissue perfusion. Replacement of electrolytes, management of acid-base disturbances, and detection and correction of underlying cause is imperative to improve outcomes.

38
Q
  1. Which of the following answers is true concerning the serologic evidence of perinatally transmitted cytomegalovirus (CMV) and the timing of the CMV infection?
    A. THe incubation period of CMV is 7-10 days
    B. 90% of women with CMV-specific immunoglobulin M (IgM) antibody will have a primary CMV infection
    C. High immunoglobulin G (IgG) avidity indicates infection occurred at least 4 months previous
    D. Routine serologic screening of pregnant women for CMV is recommended by ACOG
A

C. The incubation period of CMV is 28–60 days. CMV then induces immunoglobulin M (IgM) antibodies followed by CMV- specific immunoglobulin G (IgG) antibodies. In 2–4 months, low avidity IgG antibodies are produced. If a patient has positive IgM antibodies with low avidity IgG antibodies the CMV infection is assumed to have occurred 2–4 months prior. CMV-specific IgM is useful but not completely reliable indicator of primary infection, because just 10–30% of women with positive IgM will have a primary CMV infection. Avidity is the measure of the maturity of the IgG antibody. If the avidity is high, then it is presumed that the infection occurred at least 4 months previously. Recent infections will have IgG with low avidity.

Routine screening of pregnant women for CMV is NOT recommended by ACOG. The limitations of IgM antibodies screening along with lack of proven treatment to prevent CMV transmission diminishes the potential benefit of universal CMV screening.

ACOG Practice Bulletin #151, June 2015 “CMV, Parvovirus B19, Varicella Zoster, and Toxoplasmosis in Pregnancy.”

39
Q
  1. Which of the following answers is true concerning the serologic evidence of perinatally transmitted cytomegalovirus (CMV) and the timing of the CMV infection?
    A. THe incubation period of CMV is 7-10 days
    B. 90% of women with CMV-specific immunoglobulin M (IgM) antibody will have a primary CMV infection
    C. High immunoglobulin G (IgG) avidity indicates infection occurred at least 4 months previous
    D. Routine serologic screening of pregnant women for CMV is recommended by ACOG
A

C. The incubation period of CMV is 28–60 days. CMV then induces immunoglobulin M (IgM) antibodies followed by CMV- specific immunoglobulin G (IgG) antibodies. In 2–4 months, low avidity IgG antibodies are produced. If a patient has positive IgM antibodies with low avidity IgG antibodies the CMV infection is assumed to have occurred 2–4 months prior. CMV-specific IgM is useful but not completely reliable indicator of primary infection, because just 10–30% of women with positive IgM will have a primary CMV infection. Avidity is the measure of the maturity of the IgG antibody. If the avidity is high, then it is presumed that the infection occurred at least 4 months previously. Recent infections will have IgG with low avidity.

Routine screening of pregnant women for CMV is NOT recommended by ACOG. The limitations of IgM antibodies screening along with lack of proven treatment to prevent CMV transmission diminishes the potential benefit of universal CMV screening.

ACOG Practice Bulletin #151, June 2015 “CMV, Parvovirus B19, Varicella Zoster, and Toxoplasmosis in Pregnancy.”

40
Q
  1. A woman presents to your office with end stage renal disease and was scheduled to start dialysis but recently found out she was pregnant. When you discuss with her the pros/cons of peritoneal dialysis versus hemodialysis, which of the following do you discuss as an advantage of peritoneal dialysis over hemodialysis?
    A. Less acute fluctuations in blood pressure
    B. Less risk of infection complications
    C. Ease of catheter replacement in the third trimester
    D. Higher rates of fetal survival
A

A. Dialysis refers to renal replacement therapy that serves to correct electrolyte imbalances and remove excess fluid and toxic products of metabolism from the body. Options for dialysis include hemodialysis and peritoneal dialysis, with continuous ambulatory peritoneal dialysis (CAPD), continuous cycling peritoneal dialysis (CCPD), and nocturnal intermittent peritoneal dialysis all a part of the latter. Hemodialysis requires a vascular access for extracorporeal therapy, typically from an ateriovenous (AV) shunt. Heparinization is typically used throughout the dialysis treatment. For peritoneal dialysis, a surgically placed peritoneal catheter allows for repeated access to the peritoneal cavity. Both types of dialysis have been used successfully during pregnancy. Early reports favored peritoneal dialysis in terms of fetal survival, however the National Registry for Pregnancy in Dialysis Patients (NPDR) reported similar fetal survival rates of 39.5% for hemodialysis and 37% in peritoneal dialysis in more than 180 pregnancies. The steady state removal allowed in peritoneal dialysis offers it some clear advantages. These include a stable biochemical environment, less acute fluctuations in blood pressure (correct answer), allowing liberal fluid intake, less severe anemia. There is also no anticoagulation required for peritoneal dialysis. The peritoneal catheter can be used to give intraperitoneal insulin or magnesium, should either of these be necessary. Although there are many advantages to peritoneal dialysis, there are also specific risks, including peritonitis, catheter obstruction, and laceration of uterine vessels on placement. Additionally, hemodialysis is less labor intensive for the patient and is easier in late gestation, given the size of the uterus. If one should require a cesarean delivery, resuming dialysis after cesarean is easier and quicker while using hemodialysis.

Belfort MA, Saade G, Foley MR, et al. Critical Care Obstetrics. In: Dialysis. 2010, 5th ed. pp 188-195.

Jefferys A, Wyburn K, Chow J, et al. Peritoneal dialysis in pregnancy: a case series. Nephrology. 2008;(13):380-383.

Redrow M, et al. Dialysis in the management of pregnant patients with renal insufficiency. Medicine. 1988;67(4):199-208.

41
Q

Which of the following is indicated for a patient with Parvovirus B19 infection and seroconversion during pregnancy?

A. Ultrasound assessment every 1-2 weeks including Doppler assessment of peak systolic velocity of the fetal middle cerebral artery for 8-12 weeks
B. Monthly ultrasound assessment to monitor fetal growth and evaluate for stigmata of fetal diseases (echogenic bowel, cerebral ventriculomegaly, intracranial calcifications, and so on)
C. Prompt amniocentesis and evaluation of amniotic fluid with polymerase chain reaction (PCR) to detect parvovirus B19 DNA
Referral for treatment with hyperimmune globulin
D. Expectant management because no treatment has found to be helpful in this setting

A

A. Parvovirus infection, also known as erythema infectiosum or “fifth disease,” is caused by a single-stranded DNA virus that is mainly transmitted through respiratory secretions and hand-to-mouth contact. Manifestations in children include a facial rash with a “slapped cheek” appearance, fever, body rash, and joint pain. Immunocompetent adults may have a reticular rash, peripheral arthropathy, or be asymptomatic. Among patients with a hemoglobinopathy, potential sequelae include aplastic anemia. Individuals can transmit the virus 5–10 days after exposure, which is before the onset of the rash. In pregnant woman exposed to parvovirus, serologic screening should be performed as soon as possible. Women who are immunoglobulin M (IgM) negative and immunoglobulin G (IgG) positive are considered immune. Women who are IgM positive (regardless of IgG status) should be monitored for potential fetal infection. Women who are IgM and IgG negative are susceptible and serologic testing should be repeated in 4 weeks and considered positive if either IgM or IgG are present. Maternal to fetal transmission rates are 17–33%. Fetal manifestations may be absent or infection can result in fetal anemia, hydrops, and stillbirth. Fetal infection can be diagnosed with amniocentesis but because many cases are clinically insignificant, noninvasive surveillance is preferred. Current recommended surveillance is ultrasound evaluation every 1–2 weeks to assess for ascites, placentomegaly, cardiomegaly, hydrops, or impaired fetal growth. Doppler assessment of the peak systolic velocity of the fetal middle cerebral artery is also recommended as an accurate screen for fetal anemia. In cases of fetal anemia, further assessment and possible intrauterine transfusion are indicated. Adverse outcomes are vanishingly rare after 8–12 weeks and routine prenatal care may be resumed.

Echogenic bowel, cerebral ventriculomegaly, and intracranial calcifications are findings that can be associated with fetal cytomegalovirus (CMV) infection. Other stigmata include abdominal and liver calcifications, hepatosplenomegaly, echogenic kidneys, ascites, microcephaly, hydrops, and growth restriction. Administration of CMV hyperimmune globulin after maternal infection as a potential means of preventing transplacental transmission is under investigation but has yet to be accepted as standard of care given a paucity of evidence.

American Congress of Obstetricians and Gynecologists, Committee Opinion #151, June 2015, “Cytomegalovirus, Parvovirus B19, Varicella Zoster, and Toxoplasmosis in Pregnancy.”

Simms RA, Liebling RE, Patel RR, et al. Management and outcome of pregnancies with parvovirus B19 infection over seven years in a tertiary fetal medicine unit. Fetal Diagn Ther 2009; 25: 373.

Creasy and Resnik. Maternal-Fetal Medicine Principle and Practice, 7th ed. chapter 51, pp. 838.

42
Q
  1. Regarding the performance of non-obstetrical surgery during pregnancy, which of the following is true?
    A. No currently used anesthetic agents have been shown to have any teratogenic effects
    B. Non-urgent surgery (ie, suspicious adnexal mass) should be performed in the third trimester)
    C. Elective surgery should be postponed until the second trimester
    D. Fetal heart rate monitoring cannot assist in maternal positioning and cardiorespiratory management
A

A. Answer choice D is incorrect because that is the goal of fetal heart rate monitoring. By monitoring the fetus for evidence of acidemia, the provider may be influenced to deliver the fetus or may be alerted to maternal repositioning, need for supplemental oxygen, and so on. Answer C is incorrect because a truly elective surgical procedure should be postponed until after delivery. If nonurgent surgery (choice B) is to be performed, the ideal time is in the second trimester to avoid the highest risk of miscarriage, which would be during the first trimester and the highest risk of preterm contractions, which would be during the third trimester. The statement in answer choice A is correct, none of the anesthetic agents are known teratogens.
ACOG Committee Opinion #474 February 2011 Nonobstetrical Surgery During Pregnancy

43
Q
  1. Which of the following statements is the strongest indication for pulmonary artery catheterization and monitoring as part of the initial management of a critically ill obstetric patient?
    A. Woman with a moderate-sized uncorrected ventricular septal defect and echocardiographic evidence of elevated pulmonary artery pressure who presents with ruptured membranes at 32 weeks gestation
    B. Woman in advanced labor with a low pulse oximetry reading of 88% and a history of an uncorrected large atrial septal defect and pulmonary hypertension diagnosed by prior right heart catheterization
    C. Woman with severe hypertension, oliguria, and pre-eclampsia currently in active labor at term
    D. Woman at 28 weeks gestation with severe sepsis associated with H1N1 influenza infection
    E. Woman transferred to your tertiary care center with pulmonary edema and preterm labor at 31 weeks gestation having received terbutaline and continuous magnesium sulfate intravenous infusion
A

B. The critical care literature has demonstrated benefits and risks associated with invasive hemodynamic monitoring by pulmonary artery (PA) catheterization, and several studies have failed to identify a clear utility of this management technique among nonspecific groups of critically ill patients. The use of PA catheterization has decreased in the United States over the past few decades, and it is now quite uncommon that obstetric management requires such invasive hemodynamic monitoring. It remains useful when noninvasive monitoring cannot explain the pathophysiology of the critically ill obstetric patient, or when the patient fails to respond to conservative medical management. All of these answer choices represent potential scenarios in which invasive hemodynamic monitoring in the form of PA catheterization may be required at some point in management. The second answer choice (B) represents the strongest indication for PA catheterization in the initial management strategy. This patient carries a confirmed diagnosis of Eisenmenger’s Syndrome with cyanosis from right to left shunting. Exact fluid management with minute-to-minute monitoring and adjustment of fluid status is warranted to keep this patient alive. The woman in answer choice A may also have Eisenmenger’s Syndrome, but studies demonstrate a large discrepancy between pulmonary artery pressures measured by echocardiography and those obtained by PA catheterization. PA catheterization may be indicated to confirm or exclude the diagnosis of pulmonary hypertension, but this may be deferred if repeat high-quality 2D echocardiography suggests lower PA pressures, the patient is hemodynamically stable without evidence of cyanosis, and the patient is not actively laboring. Severe preeclampsia with oliguria represents a challenging situation in which intravascular volume status may not be easily ascertained, but initial attempts at volume resuscitation using noninvasive or less invasive monitoring strategies such as arterial line blood pressure monitoring should be the first-line therapy. Severe sepsis from H1N1 influenza infection often leads to acute respiratory distress syndrome with the possibility for increased pulmonary artery pressures, but initial management includes early goal-directed therapy with fluid resuscitation and non- or less-invasive monitoring using continuous pulse oximetry, chest radiograph, arterial line blood pressure monitoring, and 2D echocardiography. Similarly, tocolytic-induced pulmonary edema represents a scenario in which initial management involves diuretics and cessation of the inciting agent while undergoing non- or less-invasive monitoring strategies.

Clark SL and Dildy III GA. Pulmonary artery catheterization. In Belfort M, Saade G, Foley M, Phelan J, Dildy G (eds): Critical Care Obstetrics. 5th ed., 2010. Blackwell Publishing Ltd.

44
Q

. For septic shock management, a key element and goal of therapy for the initial resuscitation phase is:
A. Blood cultures drawn within 4 hours of presentation
B. Central line placement if the patient does not respond to rapid crystalloid infusion
C. Empiric antibiotics initiated with 1 hour of diagnosis
D. Transfusion of packed red blood cells for a hemoglobin level <10 g/dL

A

C. The goals of therapy for the initial resuscitation phase for septic shock include blood cultures drawn within 1 hour of presentation. Every patient diagnosed with septic shock should have central line placement within the first 4 hours of therapy. This is NOT dependent on whether the patient responds to rapid crystalloid infusion. Empiric antibiotics are to be initiated within 1 hour of presentation, and they should not be delayed while awaiting cultures. Survival differences are seen if there is a delay of only 1 hour of antibiotic therapy. Transfusion of packed red blood cells should be given for a hemoglobin level <7g/dL.
Obstet Gynecol 2012;120(3):689-706.
Albright CM, Ali TN, Lopes V, et al. The Sepsis in Obstetrics Score: a model to identify risk of morbidity from sepsis in pregnancy. Am J Obstet Gynecol. 2014;211:39.e1-8.

45
Q

The following are true regarding women with spinal cord injury in pregnancy EXCEPT:
A. Autonomic dysreflexia most commonly affects women with injury at level T6 and above
B. Autonomic dysreflexia can be precipitated by constipation, overdistended bladder, and pelvic examinations
C. During autonomic dysreflexia, antihypertensives should be avoided because of reflexive tachycardia
D. Epidural anesthesia should be started at the onset of labor to prevent autonomic dysreflexia

A

C. Autonomic dysreflexia affects >80% of patients with lesions above T6. This is caused by loss of hypothalamic control over sympathetic spinal reflexes, usually precipitated by painful stimuli, distension of bladder/bowel/uterus, or pelvic/cervical examinations. Symptoms include systemic hypertension, hyperthermia, respiratory distress, bradycardia or tachycardia, seizures, and possibly death. Antihypertensive therapy should be started immediately if there is concern for autonomic dysreflexia. Epidural anesthesia should be administered at the start of labor to prevent autonomic dysreflexia. Spontaneous vaginal delivery is possible in women with spinal cord injury, but a significant portion of these women will deliver via cesarean section.

46
Q

A 25 year old G2P0010 with a history of Marfan syndrome presents for preconception counseling. She is currently asymptomatic, and her most recent magnetic resonance imaging (MRI) demonstrates an ascending aorta measuring 51 mm in diameter. What do you recommend?
A. Okay to proceed with pregnancy
B. Start a beta blocker prior to pregnancy
C. Start an angiotensin converting enzyme (ACE) inhibitor prior to pregnancy
D. Have the ascending aorta surgically repaired prior to pregnancy

A

D. Marfan syndrome is an autosomal dominant condition with variable expressivity. The condition is characterized by a defect in connective tissue particularly fibrillin. This weakness can especially be seen in the aorta, increasing the risk of complications such as aortic dissection. Women whose aortic root is >40 mm are at greatest risk of death during pregnancy. Risk of dissection with an aortic root <40 mm is 1% and approximately 10% if it is >40 mm. If the aortic root diameter is >50 mm, elective aortic root repair/replacement should be recommended prior to pregnancy. During pregnancy goals of treatment are to decrease the pulsatile forces on the aortic with the use of β-blocker.
Women with an aortic root > 45 mm should be advised against pregnancy and those between 40–45 mm should be individualized. This patient has a family history of dissection that should be taken into account as well when counseling the patient regarding risk, in addition to her enlarged aorta. If this particular patient were to decide to proceed with pregnancy, initiation of beta-blocker therapy and cesarean delivery should be recommended as well as regular aortic imaging during pregnancy to monitor the size of her aorta.

47
Q

A 30 year old G0 presents for preconception counseling. Her medical history is complicated by a mechanical aortic valve (St. Jude). She is currently on warfarin 3 mg daily. Which of the following regimens minimizes her risk of valve thrombosis?

A. Continue international normalized ratio (INR) adjusted warfarin
B. Stop warfarin and start weight based low molecular weight heparin
C. Stop warfarin and start unfractionated subcutaneous heparin 5000 units twice daily
D. Stop warfarin and begin subcutaneous heparin 7500 units twice daily

A

A. Pregnancy in women with mechanical heart valves present a unique set of problems. Mechanical valves are associated with an increased risk of thromboembolism and valve thrombosis during pregnancy. Therapeutic anticoagulation throughout pregnancy is essential to reduce the risk of complications.
Warfarin is the most effective anticoagulant to prevent valve thrombosis and thromboembolism (2–4% risk of venous thromboembolism [VTE] in pregnancy). However, it has been associated with increased fetal anomalies and loss when used in the first trimester. The incidence of fetal embryopathy with use of warfarin in the first trimester is between 4–10%. Studies have shown that the rate of loss and anomalies are lower when the dosage of warfarin is less than 5 mg. Although unfractionated heparin does not cross the placenta or cause direct fetal effects, it is associated with higher rates of VTE during pregnancy (25%) or during the first trimester (9.2%). The 2014 American Heart Association/American College of Cardiology do not recommend the use of subcutaneous unfractionated heparin during pregnancy in women with mechanical valves.
Low molecular weight heparin (LMWH) also does not cross the placenta or affect the fetus. It does have advantages over unfractionated heparin including less risk of thrombocytopenia and more predictable therapeutic levels. Titrating LMWH levels to anti-Xa levels may improve the effectiveness of this regiment for use in these patients. The risk of thromboembolic complications during pregnancy is approximately 12%. This may be related to poor compliance with the regimen or suboptimal anti-Xa levels. Warfarin may be continued until the patient conceives because this minimizes her risk of valve thrombosis. After conception there are 2 reasonable strategies. The first is to continue the warfarin throughout pregnancy with careful monitoring of INR and adjustment as needed. The second is to transition to LMWH with a positive pregnancy test but to restart warfarin at the end of the first trimester. In either case, the patient should be counseled on the risks and consequences of both valve thrombosis versus warfarin embryopathy.

48
Q

Which of the following is true in regards to the diagnosis and treatment of pheochromocytoma in pregnancy?

A. The laboratory findings (urine and blood) are the same as gestational hypertension
B. Pregnancy affects the catecholamine levels, making diagnosis more difficult
C. Surgical excision is the treatment of choice and should occur promptly regardless of gestational age
D. Computed tomography scan is the preferred imaging for locating adrenal adenomas

A

C. A pheochromocytoma is an adrenal adenoma that secretes catecholamines, which in turn cause the signs and symptoms of the disease. These include hypertension, orthostatic hypotension, headache, abdominal and chest pain, and palpitations. The signs and symptoms of pheochromocytoma can mimic those of severe gestational hypertension, however pheochromocytoma does not have coexisting proteinuria, hyperuricemia, or edema that is present with preeclampsia.

Pregnancy does NOT affect catecholamine levels and therefore the diagnosis can be made in pregnancy. The diagnosis is made by determining the levels of metanephrines and catecholamines in a 24-hour urine collection and in the plasma. Before collection of urine and blood, methyldopa and beta blockers need to be discontinued because these medications may affect the levels. Magnetic resonance imaging of the adrenal glands is recommended to localize the tumor. The tumor is bilateral in 10% of cases. The incidence of malignancy is 10% and can only be diagnosed when metastases are present. The fetal loss rate is 50% when pheochromocytoma complicates pregnancy. If the pheochromocytoma goes unrecognized in pregnancy, the maternal mortality rate is at least 50%. The treatment for pheochromocytoma is surgical excision. Prompt surgical removal is recommended regardless of the gestational age. Preoperatively the patient should receive alpha blockade medication with pheoxybenzamine and/or phentolamine. Beta blockers should not be used until alpha blockade has been given.

49
Q

Which of the following is the MOST effective strategy to prevent neonatal abstinence syndrome (NAS) in pregnant mother taking methadone?
A. Transition from methadone to buprenorphine
B. Reduction in methadone dose
C. Discontinuation of methadone
D. Breastfeeding after delivery
E. Neonatal pharmacologic treatment after delivery

A

C. The incidence and severity of neonatal abstinence syndrome (NAS) is greater in infants exposed to methadone compared with those exposed to buprenorphine. However, NAS is common after prolonged antenatal exposure to any type of opioid. Buprenorphine may be used as a first-line therapy for pregnant opioid-dependent women who are new to treatment. The relationship between maternal methadone dose and the incidence and severity of NAS is unclear, with contradictory study results. The lack of consensus may reflect differences in study populations, different approaches to antenatal management, and differences in maternal methadone metabolism. Maternal use of methadone and buprenorphine results in low concentrations in human breast milk. Breastfeeding has been associated with less severe NAS, later development of symptoms, and reduction of pharmacologic intervention. If 1 week or longer has elapsed between the last maternal opioid use and delivery of the infant, the incidence of NAS is relatively low. Therefore, successful discontinuation of opioids before delivery is the most effective strategy to prevent NAS. However, attempts to wean or discontinue opioid maintenance during pregnancy are controversial. Case series suggest a possible increase in the risk of preterm birth, fetal distress, and stillbirth with abrupt cessation of opioids. In addition, the risk of relapse is high. For these reasons, although the risk of NAS is significant, opioid maintenance is the most common strategy during pregnancy. Any attempt to wean or discontinue opioids should be within a supervised drug treatment program. Neonatal pharmacologic therapy is indicated to relieve moderate to severe signs of NAS and to prevent complications. It is not started empirically, but rather started according to established treatment algorithms. Pharmacologic treatment is therapeutic and not preventative.

50
Q

A poorly controlled type 1 diabetic gives birth to a 10 pound infant by cesarean at 38 weeks gestation. The infant develops hyperbilirubinemia within 48 hours after delivery. The primary mechanism of hyperbilirubinemia in this infant is:
A. Increased production of bilirubin
B. Decreased clearance of bilirubin
C. Increased beta-glucuronidase activity
D. Increased enterohepatic circulation of bilirubin

A

A. Hyperbilirubinemia in infants ≥35 weeks gestation is defined as a total bilirubin >95th percentile for age. Hyperbilirubinemia is associated with an increased risk for bilirubin-induced neurologic dysfunction, which occurs when bilirubin crosses the blood-brain barrier and is incorporated into brain tissue. The term “kernicterus” is used to describe the chronic and permanent neurologic sequelae of this process. A basic understanding of bilirubin clearance and excretion is required to understand nonpathologic neonatal jaundice and also pathologic hyperbilirubinemia. Clearance and excretion of bilirubin occurs in the following ways: Hepatic uptake – Circulating bilirubin, bound to albumin, is transported to the liver. Bilirubin dissociates from albumin and is taken up by hepatocytes, where it is processed for excretion. Conjugation – In hepatocytes, uridine diphosphogluconurate glucuronosyltransferase (UGT1A1) catalyzes the conjugation of bilirubin with glucuronic acid. Conjugated bilirubin, which is more water-soluble than unconjugated bilirubin, is excreted in bile. Biliary excretion – Conjugated bilirubin is secreted into the bile in an active process and is then excreted into the digestive tract. Conjugated bilirubin cannot be reabsorbed by the intestinal epithelial cells. It is broken down in the intestine by bacterial enzymes and, in the adult, is reduced to urobilin by bacterial enzymes. However, at birth the infant’s microbiome is immature. Therefore, little if any conjugated bilirubin is reduced to urobilin. In addition, infants have beta-glucuronidase in the intestinal mucosa, which deconjugates the conjugated bilirubin. The unconjugated bilirubin can be reabsorbed through the intestinal wall and recycled into the circulation, a process known as the “enterohepatic circulation of bilirubin.” Nonpathologic neonatal jaundice is caused by normal neonatal variations in bilirubin metabolism that result in mild unconjugated bilirubinemia. Increased bilirubin production – Term newborns produce 2‒3 times more bilirubin than adults because they have more red blood cells (hematocrit between 50‒60%) and fetal red blood cells have a shorter life span than those in adults. The increased turnover of more red blood cells produces more bilirubin. Decreased bilirubin clearance – This is mainly caused by the deficiency of the enzyme UGT1A1. UGT activity in term infants at 7 days of age is approximately 1% of that of the adult liver and does not reach adult levels until 14 weeks of age. Increased enterohepatic circulation of bilirubin – This further increases bilirubin load through the mechanisms described above. Pathologic hyperbilirubinemia is caused by exaggeration of mechanisms that cause neonatal jaundice or by pathologic conditions that increase bilirubin production, decrease bilirubin clearance, or increase enterohepatic circulation. Examples include: Increased production – The most common cause of pathologic hyperbilirubinemia is increased bilirubin production due to hemolytic disease processes.

o Immune-mediated hemolysis (eg, ABO or RhD blood incompatibility)
o Inherited red blood cell membrane defects (eg, hereditary spherocytosis)
o Erythrocyte enzyme defects (eg, glucose-6-phosphate dehydrogenase [G6PD] deficiency)
o Neonatal sepsis
o Polycythemia (eg, macrosomic infants of diabetic mothers)
o Sequestration of blood within a closed space (eg, cephalohematoma).

Decreased clearance – Inherited defects in UGT1A1 decrease bilirubin conjugation and therefore reduce clearance (Crigler-Najjar and Gilbert’s Syndrome). Increased enterohepatic circulation – The major causes are breast milk jaundice and impaired intestinal motility or anatomic obstruction. Breast milk jaundice appears to be caused by a factor in human milk, yet to be identified, that promotes increased intestinal absorption of bilirubin. However, in breastfed infants, pathologic hyperbilirubinemia is more likely caused by failure to successfully breastfeed (resulting in hypovolemia) than a direct effect of breast milk itself. Of the above mechanisms, increased bilirubin production is the most likely mechanism of jaundice in the infant of a diabetic mother. Polycythemia (hematocrit >65%) occurs in 5‒10% of infants of diabetic mothers. The mechanism for polycythemia may be increased erythropoietin concentrations caused by hyperglycemia and associated chronic fetal hypoxemia. Polycythemia results in increased hemolysis and therefore increased bilirubin production. Decreased bilirubin clearance may also play a more minor role in this situation.

51
Q
  1. A cesarean delivery is performed at 33 weeks gestation due to preterm labor and breech presentation. After the initial resuscitation, the neonate has persistent tachypnea, grunting, and cyanosis and continuous positive airway pressure (CPAP) is initiated. The chest radiograph shows low lung volumes, a diffuse reticulogranular pattern and air bronchograms. Which of the following is the most likely diagnosis?

A. Transient tachypnea of the newborn (TTN)
B. Bronchopulmonary dysplasia (BPD)
C. Respiratory distress syndrome (RDS)
D. Pulmonary hypoplasia
E. Pneumothorax

A

C. Transient tachypnea of the newborn (TTN) is a mild, self-limiting condition affecting term and late preterm infants. It is classically seen in infants delivered near term, especially after cesarean birth. Clinical features include tachypnea, grunting, nasal flaring, mild intercostal and subcostal retractions, and mild central cyanosis. The chest radiograph shows prominent perihilar streaks that represent engorged pulmonary lymphatics and blood vessels. The radiographic appearance and clinical symptoms rapidly improve over 24‒48 hours. TTN is a diagnosis of exclusion and other causes of respiratory distress should be excluded. TTN is unlikely to be the primary diagnosis in this preterm infant, particularly given the chest radiographic findings.
Pulmonary hypoplasia is generally secondary to extrinsic compression on the lungs or neuromuscular dysfunction. In the absence of other risk factors, pulmonary hypoplasia would be unlikely in this case. Chest radiographic findings are not consistent with pneumothorax.
The primary risk factor for respiratory distress syndrome (RDS) is prematurity. The symptoms are nonspecific, present immediately after birth, and include tachypnea, nasal flaring, subcostal and intercostal retractions, and cyanosis. The typical clinical course is progressive worsening of symptoms over 2‒3 days followed by recovery. Exogenous surfactant replacement improves oxygenation, decreases need for ventilatory support, and increases survival. Classic radiographic findings include low-volume lungs with a diffuse reticulogranular pattern and air bronchograms.
Bronchopulmonary dysplasia (BPD) is a long-term complication of RDS. It primarily occurs in very low birth weight infants. Other risk factors include oxygen toxicity, mechanical ventilation, and inflammation. The clinical diagnosis is based on the need for supplemental oxygen at 36 weeks corrected gestational age. The transition from RDS to BPD is subtle and gradual. Chest radiographic findings include bilateral haziness that reflects diffuse microatelectasis. These changes lead to ventilation-perfusion mismatch and poor oxygenation. Although the neonate presented is at risk for BPD, the time frame suggests RDS as the primary diagnosis.

Greenberg J, et al. (2009). Neonatal Morbidities of Prenatal and Perinatal Origin. In R. Creasy, R. Resnik, J. Iams (Eds.), Creasy and Resnik’s Maternal-Fetal Medicine: Principles and Practice, 6th ed. (pp 433-451). Philadelphia: Saunders Elsevier.

52
Q

A 22 year old woman delivers a baby boy at 41 weeks gestation by cesarean delivery for arrest of descent. He weighs 9 pounds at birth and has Apgar scores of 8 and 9. What is strongest risk factor for autism spectrum disorder in the child?
A. Young maternal age
B. Male sex
C. Late term gestation
D. Cesarean delivery
E. Macrosomai

A

B. Epidemiological studies have identified various associations between prenatal, perinatal, and neonatal factors and autism spectrum disorder (ASD), but none has proven necessary or sufficient for the disease to develop. These risk factors either alone, or in combination, may increase autism risk, particularly in individuals who are genetically predisposed. Advanced paternal or maternal reproductive age has been a consistent risk in epidemiologic studies. The biology is unclear, but this may be related to germ-line mutation. Young maternal age has not been associated with increased risk of autism in offspring. Large-scale population-based studies have consistently shown that males are 2‒3 times more likely to be affected by ASD compared with females. The origins of this discrepancy are uncertain but there is wide and varied speculation as to the diagnostic biases and sex differences in genetic, endocrine, epigenetic, and environmental risk factors that might contribute. Several perinatal and neonatal risk factors have been associated with increased risk of ASD. These include prematurity, breech presentation, markers of fetal distress (such as low 5-minute Apgar score), cesarean birth, small for gestational age, neonatal encephalopathy, and birth defects, among others. There is no known association with late-term or postterm pregnancy or with fetal macrosomia. The correlated occurrence of many of these complications (eg, breech presentation and cesarean birth) limits the ability to determine which factors, if any, are independently associated with autism. In the presented case, the strongest risk factor for autism is male sex. Large, prospective population-based studies are needed to identify modifiable risk factors in early life.

53
Q

Preeclampsia risk reduction with LDA

A

. One significant risk factor for pre-eclampsia is a history of pre-eclampsia in a prior pregnancy. For patients at increased risk of pre-eclampsia, there are unfortunately few known interventions that can greatly reduce this risk. However, meta-analyses of several randomized controlled trials have shown that low-dose aspirin is associated with a modest (approximately 10%) reduction in the risk of recurrent pre-eclampsia. Based on these data, the Task Force on Hypertension in Pregnancy, convened in 2013 by the American College of Obstetricians and Gynecologists, recommended that women with a history of pre-eclampsia and preterm delivery less than 34 0/7 weeks or pre-eclampsia in more than 1 prior pregnancy be administered low-dose aspirin (60–80mg) beginning in the late first trimester (Quality of evidence: Moderate; Strength of recommendation: Qualified). In 2014, the U.S. Preventive Services Task Force recommended low-dose aspirin for women considered to be at high risk for pre-eclampsia (history of pre-eclampsia, multifetal gestation, chronic hypertension, type 1 or 2 diabetes, renal disease, systemic lupus erythematosus, antiphospholipid syndrome). Low-dose aspirin appears to be most effective when started before 16 weeks. Vitamin C and E, bed rest, and heparin have not been shown to reduce the risk of recurrent pre-eclampsia. Calcium supplementation has been shown in meta-analyses to reduce the risk of pre-eclampsia, particularly those with low dietary calcium intake. It is unclear if calcium supplementation is beneficial for women at increased risk for pre-eclampsia with adequate dietary calcium intake.

American College of Obstetricians and Gynecologists; Task Force on Hypertension in Pregnancy. Obstet Gynecol. 2013;122(5):1122-1131.

54
Q

Perivability A 33 year old G1P0 at 21 weeks 4 days presents to triage with leaking of clear fluid. Preterm premature rupture of membranes (PPROM) is confirmed by nitrazine and ferning. Her estimated date of delivery is based on an ultrasound done at 8 weeks. Besides leaking, she is asymptomatic. She declines pregnancy termination. The group b streptococcus carrier status is unknown. Which intervention is most appropriate?

A. Antibiotics to enhance PPROM latency
B. Antenatal corticosteroids
C. Magnesium sulfate for neuroprotection
D. Antibiotics for group b streptococcus prophylaxis

A

A.
In 2013, a workshop sponsored by the Society for Maternal-Fetal Medicine, Eunice Kennedy Shriver National Institute of Child Health and Human Development, the Section on Perinatal Pediatrics of the American Academy of Pediatrics, and the American College of Obstetricians and Gynecologists, defined “periviable” as delivery that occurs at 20 0/7 through 25 6/7 weeks of gestation. The publication from the workshop provided guidelines regarding obstetric recommendations for women with threatened periviable birth. Although it acknowledged the paucity of reports on antibiotics for preterm premature rupture of membranes (PPROM) to enhance latency before 25 weeks, they suggest starting latency antibiotics before 22 weeks 0/7 days, if the delivery is not imminent (Grade 2C, weak recommendation with low quality evidence). Before 22 weeks the guideline recommends against antenatal corticosteroids, intrapartum antibiotics for group B streptococcus prophylaxis, tocolytics to enhance latency for potential steroid benefit, magnesium sulfate for neuroprotection, or intrapartum antibiotics for group B streptococcus prophylaxis (Grade 1A, strong recommendation, high-quality evidence).

Raju TN, Mercer BM, Burchfield DJ, Joseph GF Jr. Periviable birth: executive summary of a joint workshop by the Eunice Kennedy Shriver National Institute of Child Health and Human Development, Society for Maternal-Fetal Medicine, American Academy of Pediatrics, and American College of Obstetricians and Gynecologists. Am J Obstet Gynecol. 2014;210:406-17.

Ecker JL, Kaimal A, Mercer BM, et al. Obstetric Care Consensus Series #3: Periviable birth. American College of Obstetricians and Gynecologists and Society for Maternal-Fetal Medicine, November 2015.

55
Q

Describe the myometrial events involved in the initiation of parturition: uterine quiescence during pregnancy is maintained by progesterone primarily via signaling mediated by which of the following:
A. Oxytocin receptor
B. PR A receptor
C. CRH receptor
D. PR B receptor

A

D.
Uterine quiescence during pregnancy is maintained by progesterone primarily via signaling mediated by the type-B progesterone receptor (PR-B) in myometrial cells. Withdrawal of PR-B-mediated progesterone activity is a principal trigger for labor. One mechanism for PR-B withdrawal is by inhibition of its activity by the type-A PR (PR-A) receptor in myometrial cells.

56
Q

Estrogen’s role in myometrial contractions includes all of the following EXCEPT:
A. Inhibits COX-2 receptor
B. Upregulating myometrial gap junctions
C. Enhances transcription of genes for the oxytocin receptor
D. Enhances transcription of genes for connexin 43

A

A.
Estrogens do not themselves cause myometrial contractions however they are involved in myometrial activation. Estrogen activation is thought to transform the myometrium into a contractile phenotype by modulating the expression of a specific genes which encode contraction-associated proteins. Estrogen has been shown to up-regulate OXTR, the gene encoding the oxytocin receptor, genes encoding myometrial gap junctions such as connexin 43, and genes encoding myometrial oxytocin receptors. In addition, estrogen is known to increase the gene expression and activity of the COX-2 receptor.

57
Q

All of the following are hypothesized mechanisms by which immune tolerance to fetal/paternal antigens is promoted during pregnancy, EXCEPT:
A. Fas/FasL systems helps promote apoptosis in paternal Ag recognizing T-cells
B. T regulatory cells help suppress activity of other T cells, thereby providing protection to paternal Ag
C. The B7H1 protein within the syncytiotrophoblast, is able to inhibit lymphocytes at the maternal fetal interface
D. Suppression of indoleamine 2,3-dioxygenase (IDO) at the maternal-fetal interface

A

D.
The Fas/FasL system is thought to selectively recognize paternal Ag recognizing T-cells and program these cells for apoptosis. The B7H1 protein, which has lymphocyte inhibitory properties, is uniquely expressed on syncytiotrophoblast and, therefore, positioned to interfere with activation of lymphocytes circulating in maternal blood. Indoleamine 2,3-dioxygenase is an enzyme that degrades tryptophan. Tryptophan is necessary for T cells to proliferate.

58
Q

Describe the altered host resistance factors in pregnant women, the fetus, and the newborn. Which of the following statements is true?
A. Depletion of natural killer cells during pregnancy is protective against early pregnancy loss
B. Uterine natural killer cells have a role in normal angiogenesis in early pregnancy
C. The decidua has a more passive, supportive role to immune cells during implantation
D. Increased levels of uterine dendritic cells at the maternal fetal interface can lead to impaired implantation

A

B.
The decidua has an active role in the regulating uterine immune cells, including controlling migration of maternal T cells through the implantation site. Natural killer cells were once thought to be cytotoxic to the trophoblast, however we now know that this is not true. Depletion of NK cells can be harmful to pregnancy outcomes. Uterine NK cells are involved in mediation of angiogenesis and trophoblast invasion. Similarly, dendritic cells have an important role in influencing correct cytokine levels at the maternal fetal interface.

59
Q

Which of the following statements about myometrial differentiation throughout pregnancy is true?
A. Myometrial differentiation occurs in 5 distinct states: Quiescence, proliferation, synthetic, contractile and labor
B. In the proliferative phase, myocyte proliferation is mediated by IGF-1
C. The synthetic phase of myometrial differentiation is regulated by increasing estrogen level in early pregnancy
D. During the labor phase of myometrial differentiation, up-regulation of CAPS is regulated by progesterone withdrawal

A

B.
Myometrial differentiation occurs in 4 distinct states: Proliferation, synthetic, contractile and labor. In the proliferative phase, myocyte proliferation is mediated by IGF-1, induced by estrogen. The synthetic phase of myometrial differentiation is regulated by progesterone. During this phase there is extensive remodeling of myometrial cells. Growth of myometrium during this phase is due to cell hypertrophy. During the labor phase of myometrial differentiation, up-regulation of CAPs is regulated at the gene transcription level.

60
Q

Which of the following coagulation factors increases in pregnancy?
A. Von Willebrand Factor
B. Factor XIII
C. Factor V
D. Prothrombin
E. Protein S

A

A.
Von Willebrand Factor increases in pregnancy. Factor XIII decreases by term to 50%; Factor V initially increases but overall by term it decreases; prothrombin remains unchanged throughout pregnancy; and protein S decreases in pregnancy.

61
Q

Which of the following aspects of kidney function during pregnancy can lead to gestational proteinuria?
A. Decreased GFR in pregnancy
B. Decreased permeability of the glomerular basement membrane
C. Increase in renal size
D. Impaired of tubular absorption

A

D.
Urinary protein excretion does increase during pregnancy in a normal state due to increase in the GFR, impairment of tubular absorption and some increased permeability of the glomerular basement membrane. There is an additional increase in this permeability that allows more albumin to be filtered into the urine during cases of preeclampsia. While maternal renal size does increase in pregnancy, this has not been shown to increase levels of proteinuria in timed urinary collection.

62
Q

Which of the following is true regarding platelets in pregnancy?
A. Platelets in pregnancy remain unchanged in healthy women but decrease in women with chronic disease.
B. Platelets remain unchanged until the third trimester when they decrease.
C. Platelets in pregnancy decrease in all women during pregnancy by 20% irrespective of preexisting disease or obstetrical complications.
D. Platelets decrease early in pregnancy but return to normal by the end of the third trimester.
E. A platelet decrease is normal in pregnancy and therefore does not require further workup

A

C.
Platelet counts in all women decrease by roughly 20% beginning in the first trimester. Although mild platelet count decreases are likely physiologic, platelet counts <100,00 should be evaluated for possible other etiologies of thrombocytopenia.

63
Q

You are seeing a patient for her 6-week follow-up postpartum visit. She reports extreme fatigue. Her hemoglobin returns as 10.5g/dL. What is the next step in management?
A. This value is expected at 6 weeks postpartum. Continue taking a multivitamin.
B. This value is borderline, so it’s prudent to return in 2-4 weeks for follow-up testing.
C. This value is abnormal and warrants immediate referral to hematology.
D. This value is abnormal; the workup begins with iron studies drawn today followed by supplementation with oral iron.
E. This value is acceptable because she’s at 10.5g/dL prepregnancy.

A

D.
The patient is anemic; iron studies should be drawn and supplementation with oral iron initiated. Anemia is defined as hemoglobin < 11 g/dL in the third trimester up to 6 weeks postpartum; thus, the patient does not have a borderline value, and workup and management should begin. Having prepregnancy anemia that persists or worsens in the postpartum period does not preclude pursuing workup and management of the condition.

64
Q

In a fetus with growth restriction from placental insufficiency, which of the following is true concerning umbilical cord serum endocrine markers?
A. High insulin
B. High c-peptide
C. Low thyroid stimulating hormone
D. Low insulin-like growth factor 1 (somatomedin-C)
E. Low insulin-like growth factor binding protein

A

D.
Insulin-like growth factor-1 (IGF-1) plays a central role in cellular metabolism, proliferation, and differentiation throughout fetal development. It is particularly important for normal fetal growth. In utero cord blood, IGF-1 levels correlate with fetal weight and bone length in the second and third trimesters, as well as birth weight. Cord serum IGF-1 concentrations are low in fetal growth restriction and preterm birth. In animal models, administration of IGF-1 has therapeutic effects on fetal growth, hypoxic-ischemic brain injury as well as retinopathy of prematurity. Choice (A) is not correct because insulin is closely related to IGF-1 and is decreased in fetal serum in cases of FGR and placental insufficiency. Choice (B) is not correct because C-peptide mirrors endogenous insulin production. Choice (C) is not correct because TSH is unchanged in FGR from placental insufficiency. Cord blood levels of free T4 and free T3 are reduced in FGR, however. Choice (E) is not correct because IGF binding protein (IGFBP) counters bioactivity of IGF-1. IGFBP is high when circulating insulin and IGF-1 are low.

65
Q

Pregnancy is considered a prothrombotic state because of adaptations to the coagulation system. Which of the following adaptations is consistent with pregnancy?
A. Increases in fibrinogen; factors II, VII, VIII, X, and XII; and von Willebrand factor with a concurrent decrease in protein S and antithrombin
B. Increases in fibrinogen; factors II, VII, VIII, X, XII; von Willebrand factor; protein S; and antithrombin
C. Decreases in fibrinogen; factors II, VII, VIII, X, and XII; von Willebrand factor; protein S; and antithrombin
D. Decreases in fibrinogen; factors II, VII, VIII, X, and XII; and von Willebrand factor with a concurrent increase in protein S and antithrombin
E. No change in clotting factors; only reduced fibrinolysis

A

A.
The prothrombotic state of pregnancy includes increases in procoagulants including fibrinogen; factors II, VII, VIII, X, and XII; and von Willebrand factor as well as a decrease in anticoagulant activity including protein S and antithrombin. Additionally, there is resistance to Protein C. Therefore, Answer (A) is correct because it accurately reflects both the increases and decreases that occur. Answers (B) and (C) show all procoagulant and anticoagulant activity increasing or decreasing. Answer (D) is incorrect because while reduced fibrinolysis does occur, changes in the balance between procoagulants and anticoagulant pathways are an important aspect of the prothrombotic state observed in normal pregnancy